[go: up one dir, main page]

0% encontró este documento útil (0 votos)
25 vistas58 páginas

Reto Enarm Preguntas Ali

El documento presenta una serie de casos clínicos que abordan diferentes diagnósticos y tratamientos en pacientes de diversas edades. Se mencionan condiciones como la enfermedad de Osgood-Schlatter, quiste pilonidal, apendicitis aguda, y complicaciones como la atrofia testicular, entre otros. Cada caso incluye detalles sobre la evaluación, diagnóstico y manejo recomendado para cada paciente.

Cargado por

aliccast18
Derechos de autor
© © All Rights Reserved
Nos tomamos en serio los derechos de los contenidos. Si sospechas que se trata de tu contenido, reclámalo aquí.
Formatos disponibles
Descarga como DOCX, PDF, TXT o lee en línea desde Scribd
0% encontró este documento útil (0 votos)
25 vistas58 páginas

Reto Enarm Preguntas Ali

El documento presenta una serie de casos clínicos que abordan diferentes diagnósticos y tratamientos en pacientes de diversas edades. Se mencionan condiciones como la enfermedad de Osgood-Schlatter, quiste pilonidal, apendicitis aguda, y complicaciones como la atrofia testicular, entre otros. Cada caso incluye detalles sobre la evaluación, diagnóstico y manejo recomendado para cada paciente.

Cargado por

aliccast18
Derechos de autor
© © All Rights Reserved
Nos tomamos en serio los derechos de los contenidos. Si sospechas que se trata de tu contenido, reclámalo aquí.
Formatos disponibles
Descarga como DOCX, PDF, TXT o lee en línea desde Scribd
Está en la página 1/ 58

Niño de 12 años es valorado por dolor en rodilla derecha de inicio progresivo y aumento de volumen

de la tuberosidad anterior tibial. El dolor aparece cuando realiza actividad física como correr, saltar y
subir escaleras y remite al reposo. EF: edema, eritema y dolor en la tuberosidad tibial. Arco de
movimiento de la rodilla completo, ausencia de inestabilidad tibial.

1.- ¿Cuál es el diagnóstico más probable?


Enfermedad de Osgood-Schlatter
2.- ¿Cuál es el tratamiento indicado?
Limitar actividad física

Mujer de 27 años llega a urgencias por dolor intenso, aumento de la temperatura y salida de material
purulento extremadamente fétido en región sacrococcígea. Hace una semana, estuvo en bicicleta
durante un largo periodo de tiempo. EF: afebril, eritema, edema e induración la región
sacrococcígea.

3.- ¿Cuál es el diagnóstico más probable?


Quiste pilonidal
¿Cuál es el tratamiento indicado?
Drenaje abierto y marsupialización

Mujer de 75 años previamente sana. Inició padecimiento actual hace dos días con dolor abdominal en
el hipogastrio y fiebre. A la exploración: TA 100/60 mm Hg, FC 76 lpm, FR 22 rpm, Temp 39 ° C, dolor
a la palpación profunda en hipogastrio con irritación peritoneal, Rovsing negativo. Leucocitos 18, 650,
Neu 90%, bandemia 10%, creatinina 1.3, examen general de orina normal.

5.- ¿Cuál es el siguiente paso en el abordaje diagnóstico?


TAC de abdomen

Se recibe en urgencias una paciente de 9 años con dolor en hemiabdomen izquierdo y en cara
anterior del hombro ipsilateral. La paciente sufrió un traumatismo contuso en el abdomen mientras
practicaba un deporte. EF: palidez, diaforesis, hipotensión y taquicardia; dolor abdominal intenso que
se agudiza a la palpación profunda y a la descompresión.

6.- ¿Contra qué microorganismos se debería vacunar al paciente?


H influenzae B y S pneumoniae

Caso 5
Paciente de 14 años se encuentra en observación por dolor intenso en el cuadrante superior izquierdo
abdominal y cara anterior del hombro ipsilateral. El dolor había ocurrido hace un par de días después
de sufrir un traumatismo contuso en un deporte de contacto. EF: hipotensión, taquicardia, palidez
tegumentaria. Abdomen doloroso a la palpación, rebote positivo.

7.- ¿Cuál es diagnóstico más probable?


Ruptura esplénica

Caso 6

Hombre de 60 años, con diagnóstico de enfermedad diverticular complicada, se encuentra febril y


con dolor abdominal en fosa iliaca izquierda. El paciente está en su quinto día de tratamiento con
ceftriaxona y metronidazol. A su ingreso, la tomografía mostró un absceso sigmoideo complejo de
4.5cm de diámetro adherido a asas adyacentes. Sin embargo, dada la situación clínica, se solicita
nueva tomografía. El reporte incluye engrosamiento mural, secuestro de la grasa adyacente y poco
aire y líquido libres.

8.- ¿Qué marcador inflamatorio es más preciso para apoyar el diagnóstico de esta
enfermedad?
Proteína C reactiva

Paciente de 13 años se encuentra en urgencias por dolor testicular intenso precedido por cefalea,
fiebre y astenia. El dolor se ha agudizado en las últimas horas, irradia hacia la ingle y le causó
vómito. El paciente tiene vida sexual activa y ha tenido múltiples parejas en el último año. EF:
taquicardia, testículo derecho más próximo a la ingle que el contralateral, reflejo cremasteriano
ausente y Prehn negativo.

9.- ¿Cuál es el tratamiento indicado?


Quirúrgico y analgesia

Caso 8

Niño de 7 años consulta por cefalea, ojo rojo y visión borrosa de objetos cercanos. Desde hace
tiempo, el paciente se queja de ver imágenes distorsionadas o borrosas, esto ha comprometido su
rendimiento escolar. EF: inspección ocular, fondo de ojo y movimientos oculares normales. AV 20/40
en ambos ojos.

10.- ¿Cuál es el error refractivo más frecuente en México?


Astigmatismo
Caso 9

Mujer de 25 años, previamente sana. Acude porque en exámenes de rutina ha tenido bacterias en
orina, niega síntomas irritativos urinarios. Exploración física normal. Examen general de orina con
bacterias +++, leucocitos ++, nitritos positivos, urocultivo con 1´000,000 UFC con Escherichia coli

11.- ¿Cuál es el diagnóstico?


Bacteriuria asintomática

Mujer de 27 años sufre una caída desde su propio plano de sustentación mientras tenía a su hijo
entre los brazos. Al caer, el impacto ocurrió sobre la región del carpo-metacarpo de la palma en la
mano derecha. En consulta se observa excoriación en la zona, edema y eritema, además de dolor en
la tabaquera anatómica.

12.- ¿Qué manejo recomendaría?


Colocación de yeso

Niña de 10 meses se encuentra en control del niño sano. Tiene antecedente familiar (materno) de
retinoblastoma. La evaluación de crecimiento y desarrollo, las pruebas de Brückner, Hischberg y la
evaluación oftalmológica son normales.

13.- ¿Cómo se clasifica el riesgo ocular del paciente?


Alto
14.- ¿Cuál es la siguiente acción a realizar?
Referir al oftalmólogo
15.- ¿Qué pruebas le fueron realizadas durante la consulta?
Reflejo rojo y reflejo luminoso corneal

Caso 12

Mujer de 22 años, con antecedente de infecciones respiratorias de repetición, acude a urgencias por
otalgia, dolor retroauricular, otorrea derecha y fiebre de varios días de evolución. La paciente ha
recibido algunos antibióticos para las infecciones respiratorias en los últimos meses. A la exploración,
se encuentra febril, con eritema, aumento de volumen y dolor intenso a la palpación en la región
mastoidea derecha. La otoscopía es normal.

16.- ¿Cuál es el diagnóstico más probable?


Mastoiditis aguda
Caso 13

Hombre de 23 años llega a urgencias por dolor abdominal intenso en cuadrante inferior derecho,
vómito y anorexia. El paciente ingirió un antiespasmódico y algunos analgésicos, pero el dolor no
mejoró. A le exploración, se encuentra febril, con hiperestesia en fosa iliaca derecha, signos de
McBurney y Rovsing positivos.

17.- ¿Qué diagnóstico es más probable?


Apendicitis aguda

Hombre de 62 años, ha tenido tres cirugías abdominales por trauma abdominal, colecistectomía y
apendicectomía. Acude por un cuadro de dos días de evolución caracterizado por náusea, vómito de
contenido fecaloide y constipación. A la exploración: TA 120/70 mm Hg, FC 88 lpm, FR 16 rpm,
abdomen con distensión y peristalsis de lucha, no hay datos de irritación peritoneal. Na 138, K 3.9, Ca
9.4, Mg 2.4. Radiografía de abdomen con niveles hidroaéreos.

18.- ¿Cuál es el diagnóstico más probable del cuadro abdominal?


Obstrucción intestinal

Caso 15

Durante el pase de visita, un paciente de 6 años, posoperado de apendicectomía abierta, se queja de


dolor y no contiene el llanto. A la exploración, es posible distraerlo un poco, pero mantiene las
piernas en flexión y las manos sobre la incisión. Las indicaciones incluyen paracetamol con horario y
calculado por peso por razón necesaria

19.- ¿Qué fármaco está indicado para el manejo del dolor que presenta el paciente?
Morfina

Caso 16

Mujer de 20 años se encuentra en urgencias después de sufrir un traumatismo contuso en el


cuadrante superior izquierdo abdominal. La paciente fue estabilizada con soluciones IV y se redujo el
dolor con posicionarla en decúbito. La biometría hemática reporta hemoglobina, leucocitos y
plaquetas dentro de valores de referencia. En la tomografía, se evidencia un hematoma esplénico
subcapsular mayor al 50% del órgano.

20.- ¿Cuál es el manejo apropiado?


Vigilancia intrahospitalaria
Caso 17

Hombre de 50 años refiere dolor intenso tipo cólico en la región lumbar izquierda, de inicio súbito,
acompañado de náuseas y vómito en varias ocasiones. El dolor ha ocurrido ya dos veces; la primera
sucedió al final de la micción y fue muy intenso. A la exploración, los signos vitales son normales, el
abdomen resulta doloroso a la palpación profunda en cuadrante inferior izquierdo, Giordano bilateral
negativo.

21.- ¿Cuál es el diagnóstico más probable?


Litiasis ureteral
22.- ¿Cuál es el tratamiento definitivo?
Ureteroscopía

Caso 18

Hombre de 22 años es sometido a varicocelectomía programada. La cirugía implicó una incisión


horizontal de 2cm en la región inguinal izquierda, tuvo una duración de 4 horas con sangrado mínimo
y finalizó sin complicaciones.

23.- ¿Qué tipo de cierre realizó el cirujano?


Primario
¿Cómo se clasifica la herida quirúrgica del paciente?
Limpia contaminada

Caso 19

Niña de 6 años, sin antecedentes patológicos, es traída a urgencias por dolor abdominal intenso,
intolerancia a la vía oral, llanto incoercible y fiebre. La madre observó tres evacuaciones poco
consistentes color grosella. A la exploración, se encuentra taquicárdica, pálida, irritable y en flexión.
El abdomen es doloroso a la palpación superficial y profunda.

25.- ¿Qué tratamiento recomendaría?


Enema hidrostático terapéutico
26.- ¿Qué diagnóstico es el más probable para el caso?
Invaginación intestinal

Caso 20
A 24-year-old male goes to the physician because of a painless left scrotal mass. Examination shows
a 4cm, soft, non tender mass, positive to transillumination. Auscultation is negative to bowel sounds.
Examination of the contralateral testis is unremarkable.

27.- Which is the most likely diagnosis?


Varicocele

Caso 21

Hombre de 70 años, con diagnóstico de hipertensión arterial y diabetes mellitus tipo 2, es referido a
su consulta por bacteriuria persistente y una masa palpable en fosa renal izquierda. Presenta el
reporte de una urotomografía que menciona múltiples lesiones con densidad líquida en el
parénquima renal.

28.- ¿Cuál es el diagnóstico más probable?


Pielonefritis xantogranulomatosa

Caso 22

Mujer de 25 años acude a consulta solicitando una orden para realizarse ultrasonido de hígado y vías
biliares. La paciente refiere que todas las mujeres de su familia paterna han sido intervenidas por
patología vesicular y teme padecer el mismo cuadro. Incluso, pregunta sobre la posibilidad de
someterse a una cirugía profiláctica.

29.- ¿Qué postura mencionaría sobre la cirugía profiláctica?


No se recomienda

Caso 23

A 25-year-old woman, previously healthy. She attends because her routine labs showed bacterias in
urine. She has not irritative urinary symptoms. Unremarkable physical examination. Urinalysis:
bacterias +++, white blood cells ++, nitrite positive, the culture of urine with 1,000,000 CFU of
Escherichia coli.

30.- What is the diagnosis?


Asymptomatic bacteriuria

Caso 24
A 54-year-old man with diabetes, hypertension and dyslipidemia. Presents because of a 10-month
episode of calf pain after walking 300 meters and now after walking 100 meters, the pain goes away
with rest. Physical examination: normal vital signs, bilateral cataract, decreased popliteal, tibial and
dorsal pulses.

31.- Which should be the first diagnostic test to be performed?


Ankle-brachial index

Mujer de 24 años de edad, con antecedente de hipotiroidismo, llega a urgencias por dolor abdominal
y diarrea desde hace 24 horas. El dolor abdominal comenzó en epigastrio y ahora lo localiza en
cuadrante inferior derecho. También ha vomitado un par de ocasiones. A la exploración, se encuentra
febril, en mal estado general, con dolor superficial y profundo periumbilical y cuadrante inferior.

32.- ¿Qué diagnóstico sospecha?


Apendicitis aguda

Caso 26

Mujer de 37 años, con antecedente de trastorno alimenticio, llega a urgencias por dolor hipogástrico
que se irradia a la espalda y vómito gastrobiliar en varias ocasiones. A lo largo de su padecimiento
psiquiátrico, ha tenido dolor similar en algunas ocasiones, pero ha remitido después de un par de
horas. A la exploración, se encuentra febril, con hipoestesia en cuadrante superior izquierdo.

33.- ¿Qué elemento imagenológico solicitaría?


Ultrasonido
¿Qué resultado de laboratorio apoyaría el diagnóstico?
Leucocitosis

Caso 27

Paciente de 15 años es valorado por eritema, aumento de volumen y dolor testicular desde hace tres
días. Recientemente, el dolor se intensificó e inició con fiebre. EF: taquicardia, taquipnea, diaforesis y
dolor a la palpación profunda en región inguinal y cuadrante inferior derecho abdominal. Signo de
Prehn positivo, reflejo cremasteriano presente, dolor e hipoestesia en testículo derecho. USG doppler:
aumento de vascularidad testicular.

35.- ¿Cuál es la complicación más frecuente?


Atrofia testicular
¿Cuál es el mejor siguiente paso?
Tratamiento médico
- ¿Cuál es la etiología de la enfermedad más probable?
Infección bacteriana

Caso 28

Paciente de 4 meses, con antecedente de taquipnea transitoria del recién nacido, es traído a
valoración por coloración clara cuando le da la luz en los ojos. La exploración ocular revela iris café
obscuro, reflejo gris claro, convergencia intermitente, prueba de Hirschberg y fondo normal.

38.- ¿Qué evalúa la prueba de Hirschberg?


Reflejo corneal

Caso 29
Varón de 20 años llega al servicio de urgencias con dolor abdominal incoercible que
inició hace seis horas en epigastrio y hace una hora se reubicó en el cuadrante inferior
derecho. Al interrogatorio comenta que se encuentra en ayuno desde hace seis horas
por falta de apetito y no ha tenido vómito. En la exploración física se cuantifica
temperatura en 38.3 °C, signo de Blumberg positivo y signo de Rovsing negativo y dolor
en punto de McBurney.

39.- ¿Qué prueba diagnóstica modificaría la probabilidad de la enfermedad de forma


efectiva?

Radiografía de abdomen

Caso 30

Mujer de 49 años acude a consulta por episodios recurrentes de constipación con distensión
abdominal y deposiciones oscuras y fétidas desde hace seis meses. Como antecedentes relevantes,
hace tres años se le hizo una histerectomía por carcinoma endometrial; su madre fue diagnosticada
con cáncer de ovario a los 47 años y una tía materna falleció a los 51 años por cáncer de colon. El
resultado histopatológico de una biopsia que se tomó durante una colonoscopía fue un adenoma
velloso de 4x6 cm en el sigmoides.

40.- ¿Cuál es el diagnóstico más probable?


Síndrome de Lynch tipo II
41.- ¿En dónde se describe la alteración que explica la fisiopatología de la
enfermedad?
Sistema de reparación de errores del DNA

Niño de 4 años sufre tracción del brazo con el codo en extensión y pronación. Después del evento, es
incapaz de movilizar la articulación y refiere dolor intenso.

42.- ¿Cuál es el diagnóstico más probable?


Subluxación de la cabeza del radio

Mujer de 79 años, con diagnóstico de hipertensión arterial sistémica y antecedente de valvuloplastía,


llega a urgencias por dolor abdominal súbito de intensidad máxima, primero localizado en
mesogastrio y después generalizado. A la exploración, la paciente está en mal estado, pálida,
taquicárdica, hipotensa y muy ansiosa. El precordio está arrítmico, con soplo mecánico en foco mitral;
abdomen doloroso a la palpación superficial, peristalsis hipoactiva.

43.- ¿Cuál es el tratamiento de elección?


Laparotomía inmediata

Caso 33

Hombre de 18 años llega a urgencias por dolor escrotal agudo. El padecimiento comenzó hace 3
horas, mientras sostenía relaciones sexuales. Ha tenido múltiples parejas en el último año y utiliza
condón de forma irregular. El ultrasonido testicular resulta normal, pero el examen general de orina
reporta abundantes leucocitos.

44.- ¿Cuál es el diagnóstico más probable?


Epididimitis

Caso 34

Hombre de 20 años, con antecedente de criptorquidia derecha resuelta por orquidopexia en la


infancia, acude a consulta por aumento de volumen escrotal derecho sin sintomatología agregada. La
exploración revela líquido en la bolsa escrotal e induración testicular.

45.- ¿Cuál es el diagnóstico más probable?


Seminoma

Caso 35

Hombre de 35 años refiere dolor intenso, sensación urente y aumento de volumen en región perianal.
A la exploración, se encuentra masa subcutánea pequeña, fluctuante y bien definida cerca del orificio
anal.

46.- ¿Cuál es la complicación crónica más frecuente de la enfermedad del paciente?


Fístula perianal

Caso 36

Paciente de 3 años es traída a consulta por hematuria. La paciente presenta poliaquiuria y no


controla esfínteres. La orina es color rojo, se notan coágulos pequeños. La madre refiere que ha
padecido múltiples infecciones de vías urinarias, la última vez que ocurrió fue tratada con cefuroxima
14 días.

47.- ¿Cuál es la causa más probable de las infecciones de vías urinarias en este paciente?
Reflujo vesicoureteral
48.- ¿Cuál de las siguientes opciones es la apropiada en este caso?
Referir a tercer nivel

Caso 37

Niña de 2 años es traída a consulta por desviación a la línea media del ojo izquierdo. Hace seis
meses, la madre notó opacidad pupilar, pero consideró que era irrelevante. La paciente vive en una
zona ganadera. EF: irritabilidad, dolor a la palpación ocular, percepción de luz ausente. Ojo
contralateral centrado y móvil. El USG reporta masa en cámara posterior, irregular de ecogenicidad
mayor al cuerpo vítreo y depósitos de calcio.

49.- ¿Cuál es el tratamiento indicado?


Enucleación

Caso 38

Niño de 2 años es llevado a consulta por secreción serohemática transuretral, aumento de volumen
testicular izquierdo, hiporexia, fiebre y vómito. La sintomatología genital tiene 4 meses de evolución,
pero la madre no tenía recursos para acudir a consulta. A la exploración, el testículo izquierdo se
palpa de aproximadamente 6cm de largo por 4cm de ancho, con aumento de consistencia, opacidad
a la transiluminación y difícil de movilizar. La USG reporta lesión sólida con degeneración quística en
el centro.

50.- ¿Qué linaje celular esperaría encontrar en el tumor?


Senos endodérmicos
51.- ¿Qué marcador tumoral esperaría encontrar elevado?
AFP

Caso 39

Mujer de 55 años, postmenopáusica, con antecedente de dos partos vaginales y una resección
laparoscópica de leiomioma uterino, acude a consulta por fuga involuntaria de orina. Los episodios de
fuga ocurren cuando tose, estornuda o se ríe, aunque últimamente también se han presentado
cuando siente ganas de orinar. La paciente no consume medicamentos ni tiene antecedentes
patológicos. El padecimiento ha causado aislamiento social e importante disminución de la calidad de
vida.

52.- ¿Cuál de las opciones es un factor de riesgo para la enfermedad?


Multiparidad
53.- ¿Qué procedimiento debe incluir la exploración física?
Reflejo sacro
54.- ¿Qué tipo de incontinencia presenta la paciente?
Mixta

Caso 40

Hombre de 66 años, con antecedente de diabetes mellitus tipo 2, acude a consulta por poliaquiuria,
urgencia, nicturia, tenesmo y pujo. Los síntomas se han agravado en los últimos meses y han
afectado la calidad de vida del paciente. Desde hace 12 horas, no ha podido orinar más que algunas
gotas. A la exploración, se observa y palpa globo vesical.

55.- ¿Cómo continuaría el manejo de la enfermedad?


Intervención quirúrgica programada
56.- ¿Qué padecimiento es más probable?
Hiperplasia prostática obstructiva

Caso 41
A 22-year-old male comes to the ED because of a 2-day history of intense facial pain. He has had
repetitive airway infections in the last two months, the last one 7 days ago. His symptoms include
purulent nasal discharge, painful throat, persistent cough and halitosis. Physical examination reveals
fever (39°C) shows congested nasal mucosa, hyposmia, postnasal purulent discharge, and
tenderness to palpation over both maxillae.

57.- Which drug is used as first-line therapy?


Amoxicilin-clavulanate

Caso 42

Hombre de 25 años, residente de medicina, llega al servicio de urgencias por dolor abdominal desde
hace 6 horas, ansiedad y palpitaciones. Durante el interrogatorio comenta que lleva 38 horas de
guardia y ha tomado tres latas de bebidas energizantes y aproximadamente 2 litros de café para
mantenerse despierto. Además, ha tenido poliaquiuria, poliuria y anorexia. Al explorarlo, se
encuentra con taquipnea, diaforesis y temblor en reposo.

58.- ¿Qué debe recomendarse como seguimiento?


Electrocardiografía semanal

Caso 43

Paciente de 2 años llega a urgencias por fiebre y diarrea acuosa de dos días de evolución. Está en
tratamiento con eritromicina por recomendación médica, pero el cuadro clínico persiste y la fiebre
remite con paracetamol. EF: irritabilidad, llanto sin lágrimas, fiebre, pulso y TA normales, mucosa oral
seca, faringe hiperémica, distensión abdominal leve, peristalsis audible, llenado capilar 4s.

59.- ¿Cuál es el grado de deshidratación estimado del paciente?


6-9%

Caso 44

Hombre de 72 años con índice tabáquico 100 paquetes/año, desde hace cinco años ha tenido
episodios de disnea al ejercicio y tos. Acude a urgencias por un cuadro de cuatro días de evolución
caracterizado por aumento de la tos y disnea, sin cambio en las características de la expectoración. A
la exploración: TA 130/70 mm Hg, FC 110 lpm, FR 28 rpm, caquéctico, tórax en tonel, campos
pulmonares con estertores diseminados.

60.- ¿Cuál es el tratamiento más apropiado?


Beta 2 agonistas y glucocorticoides inhalados

Caso 45

Paciente de 13 años se encuentra hospitalizado por dolor articular y edema en rodillas, episodios
vespertinos de fiebre de 38°C, eritema conjuntival, úlceras orales hemorrágicas, púrpura en
extremidades inferiores y edema escrotal. El paciente fue ingresado por dolor abdominal que
después se reportó secundario a líquido libre en cavidad. Los estudios de laboratorio reportan
pancitopenia, hipertrigliceridemia y creatinina 6.5 mg/Dl.

61.- ¿Qué prueba diagnóstica indicaría?


Biopsia renal

Caso 46

Mujer de 25 años, padece hipotiroidismo. Acude por un cuadro de cuatro meses de evolución
caracterizado por manchas en rostro y manos que han ido aumentando de tamaño y extensión. A la
exploración: dermatosis localizada a sitios comentados caracterizada por manchas hipopigmentadas
de 1 cm.

62.- ¿Cuál es el mecanismo fisiopatogénico por el que ocurre esta enfermedad?


Pérdida del funcionamiento de melanocitos
63.- ¿Cuál es el tratamiento de elección en este caso?
Esteroide tópico

Caso 47

Niña de 4 años es valorada en consulta por dermatosis polimorfa diseminada en región lumbar,
glútea y extremidades, caracterizada por pápulas, ronchas y costras hemáticas. La paciente no tiene
antecedentes patológicos ni ha recibido tratamiento previo.

64.- ¿Cuál es el diagnóstico más probable?


Prúrigo por insectos
65.- ¿Cuál es el tratamiento de elección?
Antihistamínico

Caso 48
Lactante de 5 meses es traída a urgencias por fiebre en episodios, rechazo al alimento y diarrea. La
paciente vive en condiciones insalubres y es alimentado con leche de vaca. EF: irritable, febril, peso y
talla bajos para la edad, prepucio redundante, hidratación adecuada, resto normal

66.- ¿Cuál de los siguientes datos del paciente es un criterio de ingreso hospitalario?
Probable mal apego al tratamiento
67.- ¿Cuál de las siguientes opciones es el diagnóstico más probable?
Infección de vías urinarias

Caso 49

Un hombre de 20 años acude a consulta por una dermatosis reciente de aparición abrupta. La
dermatosis consiste en parches ovales ligeramente pruriginosos con escamas en el tórax anterior. El
paciente menciona que la dermatosis comenzó con una lesión única en el abdomen hace un par de
semanas. A la exploración se nota que las lesiones en el tórax posterior siguen un patrón en árbol de
navidad.

68.- ¿Cuál es el diagnóstico más probable?


Pitiriasis rosea

Caso 50

Niño de 5 años llega a urgencias por placas blancas dolorosas en la lengua. El paciente padece asma
y se encuentra en tratamiento con inhalador. EF: eritema difuso, lesiones blancas en la superficie
lingual, desprendibles y dolorosas al raspado.

69.- ¿Cuál es la etiología más probable de la enfermedad?


C. albicans
70.- ¿Qué esperaría encontrar a la evaluación microscópica del material de las
lesiones?
Blastoconidias y pseudohifas

Caso 51

Niño de 4 años llega a urgencias por dolor agudo escrotal y fiebre de dos días de evolución. El único
evento relevante es que le fue aplicada una vacuna hace una semana. EF: febril, irritable, dificultad
para abordarlo. Prepucio redundante, edema y eritema escrotal izquierdo, reflejo cremasteriano
presente, signo de Prehn positivo.

71.- ¿Qué prueba diagnóstica se debería realizar?


Ninguna

Caso 52
Hombre de 45 años refiere tos productiva, sibilancias y disnea. Índice tabáquico 30 y antecedente de
asma en la infancia. A la EF, FC 105x’, FR 24x’, Sat02 90%. Se encuentran sibilancias de forma
bilateral que se acentúan a la espiración. Espirometría: FEV1 65%, reversible al 10% con
broncodilatador.

72.- ¿Cuál es el diagnóstico más probable?


EPOC estadio II

Caso 53

A 10-year-old boy presents to her physician with very pruritic vesicles on erythematous plaques in a
linear fashion with crusting. The rash is disseminated to his both hands and left cheek. He denies any
other symptom and does not have history of a similar rash in the past.

73.- Which is the most likely diagnosis?


Contact dermatitis

Caso 54

A 51-year-old female presents to the physician complaining of a dark-colored rash in her armpits. The
dark, thick and occasionally pruritic plaques appeared few weeks ago. Upon examination,
hyperpigmented plaques are found in the axillae and the back of the neck.

74.- Which laboratory test would be appropriate to request?


Blood glucose

Caso 55

Hombre de 50 años consulta por hiporexia, fatiga, melena y pérdida de peso de 3 meses de
evolución. El paciente no tiene antecedentes patológicos. EF: paciente somnoliento, con poca masa
muscular e hipotonía generalizada; se encuentran telangiectasias en tórax superior, ginecomastia,
hepatomegalia y eritema palmar.

75.- ¿Qué alteración esperaría tener como hallazgo de laboratorio?


Prolongación de TP

Caso 56

A 14-year-old girl presents with recurrent episodes of multiple, small, oval, oral ulcerations with
regular margins. She does not have history of previous disease or other symptoms. The physical
examination is otherwise unremarkable.
76.- Which of the following investigations would be the most useful to perform?
Full blood count and hematinic assay

Caso 57

Hombre de 37 años de bajo nivel socioeconómico. Inició padecimiento hace cuatro años con
hipopigmentación de antebrazos, brazos y piernas. A la exploración con signos vitales normales,
dermatosis en sitios comentados caracterizada por máculas hipopigmentadas con borde granular de
diversos tamaños y asimétricas, acompañadas de anestesia y anhidrosis.

77.- ¿Cuál es el agente etiológico implicado en dicha entidad?


Mycobacterium leprae

Caso 58

Mujer de 50 años, con diagnóstico de diabetes e hipertensión, refiere fiebre, sintomatología de tracto
urinario bajo y dolor lumbar de 5 días de evolución. Se diagnosticó pielonefritis aguda bacteriana y se
inició el abordaje diagnóstico y terapéutico.

78.- El estudio reportó absceso perirrenal, por tanto, ¿qué procedimiento se indica
además de continuar el tratamiento antimicrobiano?
Drenaje percutáneo

Caso 59

Hombre de 48 años refiere dolor epigástrico urente de mediana intensidad que ocurre en el
postprandio tardío. El dolor se acompaña de naúseas y regurgitación ocasional. A la exploración,
dolor a la palpación en el epigastrio, resto normal.

79.- ¿Qué prueba indicaría para el abordaje diagnóstico?


Panendoscopía

Caso 60

Hombre de 34 años consulta por úlcera sacra refractaria a tratamiento médico. El paciente tiene
paraplejia desde hace un año resultado de un accidente automovilístico. A la exploración, se
encuentra úlcera sacra de 20x15cm con exudado fétido y periferia necrótica.

80.- Si la prueba no mostrara afección ósea, ¿qué tratamiento recomendaría?


Lavado y debridación quirúrgica
81.- ¿Qué prueba solicitaría para evaluar la extensión de la infección?
Tomografía

Caso 61

Hombre de 25 años acude a consulta por dermatosis localizada en la región centrofacial,


caracterizada por eritema, telangiectasias, pápulas y pústulas.

82.- ¿Qué tratamiento indicaría?


Láser vascular

Caso 62

Niño de 7 años previamente sano. Inició hace 5 días con otalgia derecha, ataque al estado general y
fiebre. A la exploración física: Temp 37.5 °C, FC 110 lpm, abombamiento y eritema de la membrana
timpánica derecha con nivel hidroaéreo.

83.- ¿Cuál es el diagnóstico más probable?


Otitis media aguda

Caso 63

Niña de 4 años llega a consulta por fiebre, anorexia e irritabilidad desde hace 3 días. La fiebre remite
con antipirético. EF: signos vitales normales, constante manipulación del pabellón auricular izquierdo,
membrana timpánica izquierda hiperémica, abombada. Resto sin alteraciones.

84.- ¿Cuál es el factor más importante en la fisiopatología de la enfermedad?


Disfunción de la tuba auditiva

Caso 64

Neonato femenino es traído a consulta por inclinación derecha de la cabeza y mentón girado de
forma contralateral. No tiene antecedentes perinatales de importancia. La exploración física es
normal excepto por el padecimiento referido.

85.- ¿Cuál es el tratamiento indicado?


Ejercicios de estiramiento

Caso 65
Hombre de 65 años, postoperado de colectomía total con ileostomía por colitis ulcerativa crónica
inespecífica, presenta irritabilidad, fatiga, cefalalgia y náuseas.

86.- ¿Cuál es la causa más probable de la sintomatología?


Hiponatremia

Caso 66

A 37-year-old man with low socioeconomic status. Presents with a four-year history of hypochromic
areas in arms and legs. Physical examination: normal vital signs, skin lesions in previously
commented areas characterized by macular hypochromic asymmetric lesions with granular borders
and different sizes, accompanied by anesthesia and hypohidrosis.

87.- Which is the cause of the disease?


Mycobacterium leprae

Caso 67

A 3-year-old girl is brought to the ED because of high-grade fever (39.3°C) for 5 days, bilateral
conjunctivitis and a disseminated rash. Examination reveals cervical lymphadenopathy, a “strawberry
tongue”, and bilateral edema of hands and feet.

88.- Which is the most likely diagnosis?


Kawasaki disease

Caso 68

A 56-year-old woman with diabetes and hypertension. She reports a 4-year history of biliary colic,
today attends because of a one-week episode of hepatic pain and jaundice. Physical examination: BP
100/60 mm Hg, HR 110 bpm, RR 20/min temperature 38.5°C, scleral jaundice, right upper-quadrant
pain. White blood cells count 16,700, total bilirubin 2.7, direct bilirubin 2, alanin aminotransferase
234, aspartate aminotransferase 250, alkaline phosphatase 210. The ultrasound examination reveals
an hypoechoic lesion in the right lobe of the liver.

89.- Which is the most appropriate treatment?


Ceftriaxone and Metronidazole

Caso 69
Mujer de 55 años refiere dolor en extremidades superiores y dificultad para comenzar a caminar
después de estar en sedestación. Desde hace una semana, no es capaz de subir escaleras y ha
notado una dermatosis peculiar en la región cercana a los párpados. En la exploración, se corrobora
la dermatosis y se encuentran nódulos violáceos sobre los codos. Además, se evidencia limitación de
los arcos de movimientos de la articulación de la cadera.

90.- ¿Cómo se denomina la dermatosis de la paciente?


Heliotropa
¿Cuál es el diagnóstico más probable?
Dermatomiositis

Caso 70

Mujer de 79 años con lumbalgia mecánica. Tomó hace 5 horas 8 gramos de ibuprofeno. A la
exploración física: TA 90/60 mm Hg, FC 45 lpm, FR 12 rpm, agitación y convulsiones tónico-clónicas.
BT 3.5 BD 2.7 BI 0.8 ALT 560 AST 345

92.- ¿Cuál es el mecanismo fisiopatogénico por el que causó intoxicación?


Acumulación de 2-hidroxi-ibuprofeno

Caso 71

Niña de 14 meses es traída a consulta por rechazo al alimento y fiebre desde hace tres días. En la
exploración física se encuentra con temperatura de 38.6°C, irritable, reactiva, sin signos de
deshidratación. Usted sospecha que tenga una infección del tracto urinario y toma una muestra para
urocultivo por cateterismo vesical.

93.- ¿Con cuántas UFC consideraría positivo el urocultivo?


>10,000

Caso 72

A 25-year-old known atopic eczema patient currently has a nine month old son and is presenting to
your clinic complaining of generalized itchness of her skin all over her body and of painful cracked
hand skin. By examination, the patient's hands are very dry with some fissuring of her fingertips.Her
skin appears to be dry-looking with scattered poorly defined scaly macules. By checking the patient's
medical record, you see that she had repeatedly refilled her moderate topical corticosteroid cream
but did not get any emollients issued in the past six months. Your diagnosis is irritant contact
dermatitis due to contact with detergents while caring for the baby.

94.- Which of the following is the most appropriate treatment regimen?


A potent topical corticosteroid ointment for the hands, a
moderately greasy topical emollient for use on the whole body,
and an emollient wash to be used instead of soap

Caso 73

Hombre de 30 años con sinusitis crónica presenta síntomas de compromiso neurológico. Se inicia
protocolo de estudio por sospecho de meningitis bacteriana.

95.- ¿Qué podría explicar la sintomatología neurológica del paciente?


Respuesta inmune del huésped
96.- ¿Qué hallazgos espera encontrar en el LCR, además de leucocitos?
Proteínas elevadas y glucosa baja

Caso 74

Hombre de 65 años acude por una lesión color pardo, poco pruriginosa y áspera en su espalda de seis
meses de duración. En la exploración física se observa una lesión dermatológica ovalada, elevada y
pigmentada, que parece estar adherida sobre la piel.

97.- ¿Cuál es el diagnóstico más probable?


Queratosis seborreica

Caso 75

Hombre de 40 años es valorado por datos clínicos y radiológicos compatibles con empiema pleural.
Se decide realizar toracocentesis para realizar el diagnóstico.

98.- Si se confirmara el diagnóstico de empiema, ¿cuál sería la terapia indicada?


Drenaje con sonda pleural

Caso 76

Varón de 66 años con diagnóstico previo de tuberculosis pulmonar se presenta a consulta cinco días
después de la fecha programada debido a que no había quién lo llevara al centro de salud. Hace una
semana completó dos meses de tratamiento antifímico intensivo, se le realizó el estudio microscópico
del esputo y se observaron bacilos ácido-alcohol resistentes.

99.- ¿Qué se debe hacer respecto al tratamiento?


Añadir un medicamento al esquema

Caso 77

Adolescente masculino de 14 años es traído a la consulta por pápulas, pústulas y quistes


eritematosos con algunas cicatrices en el rostro desde hace seis meses. Estuvo en tratamiento con
eritromicina tópica las últimas dos semanas sin mejoría.

100.- ¿Qué tratamiento prescribiría?


Antibiótico oral y tretinoina

Caso 78

Mujer de 58 años es traída al servicio de urgencias por dolor torácico, fiebre y tos productiva. Hace
una semana completó diez días de tratamiento antibiótico intrahospitalario por neumonía adquirida
en la comunidad. Después de ser dada de alta continuó con accesos de tos, anorexia, pérdida
ponderal y fatiga. Al explorarla tiene temperatura de 38.6°C, saturación de oxígeno de 94%,
taquipnea, malestar general, mucosas pálidas y deshidratación leve. Se integra síndrome de
consolidación pulmonar en el lado izquierdo.

101.- ¿Cuál es el tratamiento indicado?


Antifímicos

Caso 79

Niña de 12 años consulta por tos de dos semanas de evolución que se ha vuelto constante y le
produce vómito. Estuvo en tratamiento con amoxicilina, pero empeoró cuando acabó el tratamiento.
La paciente cuenta con esquema de vacunación completo, habita en condiciones insalubres y de
hacinamiento. EF: hemorragia conjuntival bilateral.

102.- ¿Cuál es el tratamiento de elección?


Azitromicina

Caso 80
Mujer de 29 años acude a valoración por dermatosis diseminada a pliegues, piel cabelluda y región
sacrococcígea, caracterizada por placas eritematoescamosas elevadas, bien delimitadas con aspecto
grisáceo en la superficie. El padecimiento comenzó hace varios meses y se localizaba en una región,
la diseminación fue progresiva.

103.- ¿Cuál es la fisiopatología de la enfermedad?


Hiperproliferación de queratinocitos

Caso 81

Recién nacido masculino de 8 días es traída a consulta por fiebre no cuantificada, vómito espontáneo
y rechazo al alimento. En las últimas horas, presentó movimientos anormales. A la exploración se
encuentra febril, taquicárdica y taquipneica, con mal aspecto; fontanelas normotensas, hipertonía y
episodios de apnea. Se indica biometría hemática, hemocultivo y punción lumbar.

104.- El análisis citoquímico LCR reporta pleocitosis neutrofílica, hipoglucorraquia e


hiperproteinorraquia. ¿Cuál es la etiología más probable?
Bacteriana

Caso 82

A 25-year-old woman, thin and previously healthy. History of upper respiratory tract infection two
weeks ago which resolved without treatment. Three-day episode characterized by dyspnea, thoracic
pain and arrythmias. Physical examination: BP 110/70 mm Hg, HR 120 bpm, RR 22/min, oximetry
96%, unremarkable pulmonary examination and tachycardiac on cardiac examination. Elevated
cardiac enzymes, sinus tachycardia and ST segment depression.

105.- Which is the most appropriate treatment?


Nonsteroidal Anti-inflamatory drugs

Caso 83

Paciente de 40 años llega a urgencias por fiebre vespertina alta y dolor abdominal en cuadrante
superior derecho. El patrón de fiebre es súbito, vespertino o nocturno; el dolor se mantiene en
hipocondrio derecho, es de intensidad moderada y se agudiza al movimiento. Como antecedentes, el
paciente consume alcohol frecuentemente y ha presentado síndrome diarreico en múltiples
ocasiones. EF: febril, hepatomegalia dolorosa. Masa firme y móvil en fosa iliaca derecha.

106.- ¿Cuál es la causa más probable del dolor en fosa iliaca derecha?
Ameboma
107.- ¿Cuál es el diagnóstico más probable?
Absceso hepático

Caso 84

Hombre de 80 años sano. Acude por presentar una lesión ulcerada en el surco nasogeniano de un
año de evolución. A la exploración observas lesión de 3 cm ulcerada en el sitio descrito.

108.- ¿Cuál es el siguiente paso en su manejo?


Biopsia por sacabocado
109.- ¿Cuál es el diagnóstico más probable?
Cáncer basocelular

Caso 85

Adolescente de 15 años, con antecedente de hemólisis por incompatibilidad de grupo sanguíneo, se


encuentra en urgencias después de someterse a una ligadura de várices esofágicas. El padecimiento
comenzó con dolor epigástrico ardoroso espontáneo, hematemesis franca y melena. A la exploración,
se palpa esplenomegalia aproximadamente 7cm bajo el borde costal. Cuenta con una angioTAC
abdominal que evidencia vena porta con deformación cavernomatosa con vasos colaterales gástricos
y esplénicos.

110.- ¿Cuál es la causa más común del síndrome en población pediátrica?


Trombosis de vena porta
111.- ¿Qué síndrome clínico presenta el paciente?
Hipertensión portal

Caso 86

Masculino de 15 años sufre herida con un objeto punzocortante mientras participaba en una clase de
cocina. El paciente presenta una herida cutánea de 4 cm de longitud, con bordes regulares y
hemorragia activa leve, localizada en la región medial del antebrazo izquierdo.

112.- ¿En cuánto tiempo solicitaría cita para retiro de sutura?


7-10 días

Caso 87
A 19-year-old female arrives at the physician because of a non-painful oral ulcer and intermittent
swelling of both knees in the past three months. When the swelling occurs, she feels so much pain
that it makes her unable to walk. She does not have other symptoms or history of disease and denies
sexual activity. The physical examination is unremarkable except for BP of 150/80 mmHg. She brings
laboratory results that reveal leukopenia, thrombocytopenia, ESR 90 mm/h, ANA 1:160, RF negative,
Anti-DNA positive, and proteinuria 3+.

113.- Which of the following findings would you expect on a joint radiography?
Periarticular osteopenia

Caso 88

Mujer de 65 años, diabética y obesa. Inició hace 2 años con pústulas en ambas axilas, dolorosas,
pruriginosas, de base eritematosa, pruriginosas, que se controlan por largos periodos pero suelen
recurrir varias veces al año.

114.- ¿Cuál es el microorganismo implicado en esta infección?


Staphylococcus aureus

Caso 89

Hombre de 60 años de edad, con antecedente de tabaquismo crónico intenso, acude por la presencia
de una úlcera única, ubicada en el borde lateral derecho de la lengua, de cuatro meses de evolución.
La úlcera tiene márgenes elevados, indurados e irregulares. Además, se palpa una adenomegalia en
la región submandibular derecha.

115.- ¿Cuál es el diagnóstico más probable?


Carcinoma oral de células escamosas

Caso 90

Mujer de 25 años inició padecimiento actual hace 3 años con episodios intermitentes de
parotidomegalia, xerostomía y xeroftalmía. Acude a urgencias por un cuadro de una semana de
evolución caracterizado por disminución de la fuerza muscular en las piernas. A la exploración:
xeroftalmía, xerostomía, parotidomegalia y fuerza muscular 3/5 distal en ambas piernas con reflejos
de estiramiento muscular conservados. Gasometría arterial: pH 7.3, HC03 12, K 2, AG 8.

116.- ¿Cuál es el diagnóstico más probable?


Síndrome de Sjögren
Caso 91

Mujer de 55 años con antecedente de DM2 de 5 años de evolución refiere fiebre, escalofríos, urgencia
urinaria y orina turbia. EF: taquicardia y taquipnea, TA 100/70 mmHg, fiebre, diaforesis. Dolor
abdominal difuso a la palpación, Giordano derecho. Leucocitosis, neutrofilia y bandemia. Creatinina
1.6mg/dL, glucosa 180mg/dL. Leucocitos incontables y nitritos positivos en orina. USG renal: aumento
de ecogenicidad derecho y aumento de tamaño.

117.- ¿Cuál es el diagnóstico más probable?


Sepsis con foco urinario
118.- ¿Cuál es el tratamiento indicado?
Hospitalización y antibióticoterapia IV con cobertura para gram
negativos

Caso 92

Adolescente femenino de 14 años, acude por la aparición recurrente de múltiples úlceras orales,
pequeñas, ovales y con márgenes regulares. Sin antecedentes patológicos previos ni otros síntomas
o hallazgos relevantes en la exploración física.

119.- ¿Cuál de los siguientes es el mejor tratamiento?


Corticoesteroide tópicos

Caso 93

Hombre de 35 años acude a consulta por una úlcera solitaria en el labio superior, de tres semanas de
evolución. A la exploración, la úlcera es indolora, firme y con bordes indurados. Asimismo, la
exploración cervical muestra múltiples adenomegalias de consistencia ahulada. El paciente comenta
antecedente de relaciones sexuales sin protección con múltiples parejas sexuales.

120.- ¿Cuál es el diagnóstico más probable?


Sífilis primaria

Caso 94

Neonato de 20 días es traído a urgencias por vómito postprandial de contenido gástrico desde hace
un par de días que empeoró en las últimas horas. Ahora, después de cualquier ingesta, el paciente
vomita de inmediato. A la exploración se encuentran signos de deshidratación.
121.- ¿Qué tratamiento debe recomendarse?
Piloromiotomía

Caso 95

Mujer de 76 años con diagnóstico de diabetes desde hace 45 años, como complicaciones asociadas
tiene retinopatía proliferativa, polineuropatía sensitiva distal y neuropatía autonómica. Inició con
edema de extremidades inferiores hace un año. A la exploración física: TA 140/90 mm Hg, FC 80 lpm,
18 rpm, edema leve de extremidades inferiores. Creatinina 5, Ca 7.8, P 6,examen general de orina
con proteínas ++, albuminuria de 24 horas de 1.5 gramos

122.- ¿Cuál es el sitio más probable de la lesión glomerular?


Podocito

Caso 96

Mujer de 30 años previamente sana. Inició hace 3 meses con edema de extremidades inferiores. A la
exploración física: TA 120/70 mm Hg, FC 80 lpm, 18 rpm, anasarca. Creatinina 1.5, albúmina 1.8,
colesterol 356, triglicéridos 400, examen general de orina con proteínas ++++.

123.- ¿Cuál es el mecanismo fisiopatogénico por el que ocurre esta glomerulonefritis?


Depósito de IgG y C3 en el glomérulo

Caso 97

Hombre de 25 años es valorado en consulta por fiebre, malestar general y dermatosis localizada en el
dorso del pie derecho, caracterizada por una placa eritematosa, caliente, brillante y muy dolorosa, de
varios centímetros de diámetro y con límites bien demarcados.

124.- ¿Cuál es el diagnóstico más probable?


Erisipela

Caso 98

A 60 year-old-man presents to the emergency department with generalized abdominal pain and
malaise. The patient refers having vomited five times after his last meal. He used various medications
the past day to control nausea and diarrhea. His mouth is dry, he talks slowly because of the pain and
feels weak. His medical history is notable for a laparoscopic cholecystectomy five years ago and
gastroesophageal reflux. His vital sights include temp 36.3°C, bp 100/60 mmHg, hr 96 beats/min. His
abdominal examination reveals distention, generalized pain 7/10 in intensity upon palpation, almost
absent peristalsis. The Blumberg sign is negative. Initial lab tests reveal Hb 16.5g/dL, white blood cell
count of 8,200 with 50% neutrophils and no bands. A renal panel reveals creatinine level of 1.6mg/dl,
sodium of 126 mmol/L, potassium of 2.8 mmol/L and chloride of 98 mmol/L.

125.- Apart from IV hydration, which management is indicated?


Nasogastric tube
Caso 99

Niña de 11 años es traída a urgencias por tos productiva con esputo blanco-verdoso, fiebre de 39°C,
sibilancias audibles e hiporexia. A la exploración, la saturación de O2 es de 85%, se encuentra
matidez a la percusión en región subescapular derecha y estertores gruesos bilaterales.

126.- ¿Cuál es el diagnóstico más probable?


Neumonía adquirida en la comunidad

Caso 100

Niña de 10 años es atendida en urgencias por disnea, aleteo nasal, cianosis peribucal y tiraje
intercostal. Los padres manifiestan su preocupación porque es la tercera vez que ocurre algo similar
después de exponerse a cambios de temperatura. En los últimos meses, ha tenido accesos de tos,
hiporexia, sibilancias audibles y bajo rendimiento escolar. Su esquema de vacunación está
incompleto.

127.- ¿Cuál es el diagnóstico más probable?


Asma bronquial

Caso 101

Mujer de 27 años padece, desde hace prácticamente un año, dolor abdominal cólico, artralgias en
codos y diarrea con sangre ocasional. En los últimos dos días, el dolor abdominal aumentó de
intensidad, se localizó en el cuadrante inferior derecho y se acompañó de fiebre. EF: taquicardia,
normotensión, diaforesis. Abdomen semigloboso con peristalsis disminuida, se palpa una masa en el
cuadrante inferior derecho. Laboratorio: Hb 12.5 g/dL, glucosa 80mg/dL, leucocitosis, neutrofilia y
linfocitosis. LDH 450 UI, pANCA negativos, anticuerpos antiSaccharomyces cerevisiae positivos.

128.- ¿Cuál es el tratamiento indicado?


Aminosalicilatos y corticosteroides

Caso 102
A 25-year-old woman with hypothyroidism. Presents with a four-month history of hypopigmented
spots in face and hands which have grown since they first appeared. Physical examination: Skin
lesions located in above-mentioned areas, characterized by hypopigmented macules of 1-3 cm.

129.- Which is the preferred treatment?


Topical steroids

Caso 103

Hombre de 16 años que tiene diarrea con intolerancia a la vía oral desde hace dos días. A la
exploración: TA 100/60 mm Hg FC 110 FR 20 rpm, mucosas deshidratadas. Creatinina 2.5 con FENa
0.4%

130.- ¿Cuál es el diagnóstico más probable de la falla renal en este paciente?


Falla renal aguda prerrenal

Caso 104

Niño de 8 años es traído a consulta por fiebre, tos y malestar general. La madre comenta que en su
escuela otros seis niños se han reportado enfermos con un cuadro similar. En la exploración física se
encuentra temperatura de 40.3 °C, edema palpebral y conjuntivitis bilaterales.

131.- ¿Qué recomendaría como tratamiento inicial?


Hidratación, antipiréticos y vitamina A

Caso 105

Mujer de 35 años, con antecedente de cuadro gripal hace 10 días, refiere mialgias, artralgias, fiebre,
dolor intenso en la región anterior de cuello que aumenta con la deglución y palpitaciones. EF:
tiroides aumentada al doble de tamaño, dolorosa a la palpación, sin linfadenopatía, ligero temblor
distal.

132.- ¿Cuál es el diagnóstico más probable?


Tiroiditis subaguda
Caso 106

Hombre de 27 años acude a consulta por dermatosis diseminada en piel cabelluda, surcos
nasogenianos, alas nasales, y región cigomática caracterizada por placas eritematosas pruriginosas,
adherentes y oleosas con descamación fina.

133.- ¿Cuál es la fisiopatología de la enfermedad?


Cofactor fúngico causante de alteraciones inmunológicas
134.- ¿Cuál es el diagnóstico más probable?
Dermatitis seborreica

Caso 107

Hombre de 61 años, con antecedente de hipertensión arterial sistémica y diabetes mellitus tipo 2 de
larga evolución. Consulta por accesos de tos y hemoptisis, dolor pleurítico derecho, fiebre, diaforesis
nocturna y pérdida mayor a 10kg de peso. A la exploración, se encuentra ptosis, miosis y anhidrosis
derecha.

135.- ¿Qué diagnóstico sospecha?


Tumor de Pancoast
136.- ¿Qué prueba diagnóstica indicaría inicialmente?
Radiografía de tórax

Caso 108

Paciente de 15 años acude por tos, fiebre y faringodinia que le impide la deglución. EF: T° 39.5°C,
hiperemia gingival, exudado y múltiples lesiones ulcerosas faríngeas y ganglios de la cadena cervical
anterior dolorosos a la palpación.

137.- ¿Cuál es el tratamiento indicado?


Aciclovir

Caso 109

Neonato presenta dermatosis petequial diseminada, de forma súbita, sin manifestaciones


acompañantes. Su madre no llevó control prenatal adecuado y tuvo ruptura prematura de
membranas. También, consumió aspirina por cefalalgia tres días antes del parto. El laboratorio
reporta trombocitopenia severa y hemoglobina normal.

138.- ¿Cuál de las siguientes enfermedades es más probable que causara el cuadro
clínico?
CMV congénito
Caso 110

Mujer de 20 años previamente sana. Inició hace 4 semanas con una placa eritematosa de 3 cm en
cara anterior de tórax y hace 3 semanas rash eritematoso y pruriginoso que se extendió todo el tórax
anterior. A la exploración física: dermatosis localizada en tórax anterior caracterizado por placas
rosadas con descamación fina en la periferia.

139.- ¿Cuál es el tratamiento de elección?


Jabón neutro
140.- ¿Cuál es el diagnóstico más probable?
Pitiriasis rosada

Caso 111

Hombre de 20 años es recibido en urgencias por vómito en múltiples ocasiones, dolor abdominal y
diarrea. Se encuentra muy inquieto, no es capaz de responder al interrogatorio porque se mueve
constantemente. EF: FC 115x’, FR 20x’, 37.5°C, TA 100/60 mm Hg; midriasis, epifora, piloerección y
peristalsis audible.

141.- ¿Qué escala se utiliza para evaluar la gravedad del síndrome?


CIWA

Caso 112

Hombre de 70 años que padece hipercolesterolemia leve. Acude por tener cifras persistentes de
tensión arterial mayores a 140/90 mm Hg. A la exploración: TA 160/90 mm Hg, FC 87 lpm, FR 18 rpm
con resto de exploración irrelevante.

142.- ¿Cuál es el tratamiento de elección?


Amlodipino

Caso 113

A 82-year-old man with hypercholesterolemia, diabetes and hypertension. Presents to the emergency
room because of a one-hour episode characterized by abdominal pain without any previous
abdominal trauma. Physical examination: BP 60/30, HR 135 bpm, RR 24/min, obtunded, peritoneal
signs, as well as decreased peripheral pulse. Hemoglobin 4.2, MCV 91, WBC 6,700, Platelets 176,000.
Abdominal ultrasound shows presence of blood in peritoneal cavity.

143.- Which is the most likely diagnosis?


Abdominal aortic aneurysm rupture

Caso 114

Niño de 9 años es llevado a consulta por irritabilidad, aislamiento, mal rendimiento escolar, apatía y
falta de atención. Cuando llega a su casa, pasa el tiempo dentro de su habitación, prefiere no convivir
con las personas y duerme más de 12 horas diarias.

144.- ¿Qué patología psiquiátrica tiene alto riesgo de presentar el paciente?


Trastorno depresivo mayor
145.- ¿Qué síntomas podrían estar asociados al cuadro clínico?
Ideas suicidas

Caso 115

Hombre de 65 años es valorado en urgencias por disnea de pequeños esfuerzos y palpitaciones.


Tiene antecedente de hipertensión arterial sistémica desde hace 30 años. EF: somnolencia,
taquicardia, hipotensión, taquipnea, palidez generalizada. Estertores gruesos en ambos campos
pulmonares, ruidos cardiacos rítmicos, edema en miembros inferiores. ECG: crecimiento auricular
izquierdo y eje eléctrico desviado a la izquierda.

146.- ¿Qué prueba diagnóstica es útil para identificar las potenciales anomalías cardiacas
del paciente?
Ecocardiograma

Caso 116

Niña de 6 años es valorada por asimetría facial posterior a una infección de vías aéreas superiores y a
un traumatismo leve. EF: excoriaciones múltiples en hemicara izquierda, asimetría facial
desfigurativa, músculo frontal inmóvil, oclusión palpebral parcial y movimientos bucales desiguales al
esfuerzo.

147.- ¿Cuál es la gravedad de la parálisis según la escala de House y Brackmann?


IV

Caso 117
Hombre de 65 años que padece diabetes e hipertensión con control inadecuado. A la exploración: TA
160/100 mm Hg, FC 76 lpm, FR 18 rpm, el resto de la exploración es irrelevante. Creatinina 0.5, Na
136 K 3.8 Cl 100. Toma losartán 100 mg al día.

148.- ¿Cuál es el tratamiento indicado en este paciente para control de la hipertensión?


Losartán 100 mg/día+clortalidona 50 mg/día

Caso 118

A 22-year-old women with epilepsy diagnosed during childhood. Presents with hirsutism and gingival
hypertrophy.

149.- Which anticonvulsant is most likely to cause these effects?


Phenytoin

Caso 119

Recién nacido femenino es evaluada en urgencias tras haber nacido en la vía pública. Se recibió
cubierta en una toalla húmeda, con el cordón umbilical pinzado y cortado. La paciente tiene esfuerzo
respiratorio discontinuo, depresión del tono muscular, acrocianosis generalizada y FC de 90 lpm.

150.- ¿Qué porcentaje de eventos hipóxicos ocurren durante el parto?


0.85

Se encuentra en valoración una recién nacido, producto de parto eutócico con peso y talla adecuados
para edad gestacional, por hipoactividad e irritabilidad. A la exploración, los signos vitales son
normales, pero no es capaz de succionar con suficiente fuerza para alimentarse y se nota priapismo.
El laboratorio reporta Hb 19g/Dl, Hto 66%, glucosa 99mg/Dl, y electrolitos dentro de parámetros
normales.

151.- ¿Qué manejo recomendaría?


Monitoreo cardiorrespiratorio
152.- ¿Cuál es el diagnóstico más probable?
Policitemia neonatal

Caso 121

Hombre de 20 años, sin antecedentes patológicos, consulta por cefalalgia intensa y fosfenos de un
mes de evolución. A la exploración, TA 180/100 mm Hg y taquicardia, el resto normal.
153.- ¿Qué diagnóstico considera en su diferencial?
Hipertensión renovascular

Caso 122

A 66-year-old man with medical history of diabetes and hypertension; he has smoke for the past 25
years with a tobacco index of 20 packs/ year. He comes for a follow-up visit, denies angina-like
symptoms. Unremarkable physical examination.

154.- For which of the next diseases is the screening indicated?


Aortic aneurysm

Caso 123

Niño de 4 años es llevado a consulta por astenia y lesiones equimóticas en extremidades pélvicas sin
causa aparente. Desde hace 8 meses, ha sido hospitalizado 5 veces por infecciones de vías
respiratorias. En la exploración física, se encuentran talla y peso por debajo del 5to percentil,
dificultad para caminar, microftalmia, palidez generalizada, áreas hipopigmentadas e
hiperpigmentadas en la espalda y pulgares hipoplásicos. Los estudios de laboratorio muestran
trombocitopenia grave, leucopenia, anemia y neutropenia moderadas.

155.- ¿Cuál es la cuenta celular estimada del paciente?


Neutrófilos >500/mm3, plaquetas <20,000/mm3

Caso 124

Mujer de 45 años, con diagnóstico de Transtorno Depresivo Mayor en tratamiento con dosis máxima
de fluoxetina desde hace cuatro semanas, acude a consulta de seguimiento. La sintomatología no ha
mejorado, la paciente aún no desea salir de su habitación ni convivir con las personas. Ha tenido
ideación suicida en los últimos días.

156.- ¿Cuál es el mecanismo de acción del medicamento que consume actualmente?


Inhibidor selectivo de la recaptura de serotonina

Caso 125

Mujer de 42 años previamente sana. Inició hace dos semanas con dolor precordial y fiebre
intermitente. A la exploración: TA 120/70 mm Hg, FC 110 lpm, FR 18 rpm, frote pericárdico.
Radiografía de tórax con cardiomegalia grado I y electrocardiograma con elevación difusa del
segmento ST.
157.- ¿Cuál es el diagnóstico más probable?
Pericarditis aguda

Caso 126

A 27-year-old man is admitted to the emergency department of a psychiatric hospital after a suicidal
intent. He has made other five suicidal attempts in the last half year after arguing with his girlfriend
or parents. When interrogated, he denies being suicidal, he doesn’t want to interact with people or
answer questions. Slashes on his and forearm are found in the physical examination.

158.- Which of the following is the most likely diagnosis?


Borderline personality disorder
159.- When does the disorder usually become recognizable?
Adolescence

Caso 127

Niña de 10 años se encuentra en evaluación por asimetría facial, disestesia y epifora de lado derecho.
Los síntomas comenzaron hace 4 días, cuando notó que no podía cerrar el ojo por completo. La
parálisis es evidente en reposo, la oclusión palpebral es incompleta y no es capaz de contraer el
músculo frontal.

160.- ¿Cuál es el tratamiento que mejora el pronóstico?


Esteroide y antiviral

Caso 128

Niña de 8 años acude a consulta por astenia, fatiga y dolor generalizado después de realizar actividad
física. EF: signos vitales normales, peso 49kg, talla 137cm, respiración oral y acantosis nigricans en
cuello.

161.- ¿Cuál es el factor causante de la acantosis nigricans del paciente?


IGF
162.- ¿Qué valor de hemoglobina glucosilada sería un indicador de Diabetes
Mellitus tipo 2 en el paciente?
6.5%

Caso 129

Neonato masculino, obtenido por vía abdominal con líquido amniótico meconial, requirió ventilación
con presión positiva y se calificó con Apgar 5/6 e inicio de la respiración espontánea de forma
tórpida.
163.- ¿Cuál es la medida terapéutica a recomendar?
Ventilación de alta frecuencia

Caso 130

Mujer de 33 años sufre cefalalgia de extrema intensidad, con pérdida transitoria del estado de alerta,
tras realizar actividad física extenuante. En su valoración de urgencias, se encuentra con TA 140/100
mm Hg, FC 102x’, FR 25x’, intranquila, con rigidez de nuca severa, vómito en proyectil y edema de
papila.

164.- ¿Cuál es el diagnóstico más probable?


Hemorragia subaracnoidea

Caso 131

Hombre de 88 años que padece fibrilación auricular. Acude por un cuadro de un día de evolución
caracterizado por palpitaciones, tos y disnea de pequeños esfuerzos. A la exploración: TA 200/120
mm Hg, FC 145 lpm, FR 32 rpm, S02 82%, ruidos cardiacos arrítmicos, campos pulmonares con
estertores diseminados, edema de extremidades inferiores. Radiografía de tórax con imagen en alas
de mariposa y electrocardiograma con fibrilación auricular

165.- ¿Cuál es el diagnóstico más probable?


Edema agudo pulmonar

Caso 132

A 21-year-old man previously healthy. History of diarrhea two weeks ago, attends because of
weakness of lower extremities for the past three days. Physical exam: normal vital signs, muscular
strength 3/5, absent deep tendon reflexes in inferior extremities without bladder dysfunction.

166.- Which of the results are most likely to be found?


Cerebrospinal Fluid with white cell count of 0 and 100 mg/dl
proteins

Caso 133

Hombre de 53 años, con diagnóstico de hipertensión arterial sistémica de 10 años de evolución,


acude a consulta por palpitaciones y disnea leve. EF: TA 170/100 mmHg, FC 115x’, ansioso, pálido.
Precordio arrítmico, sin agregados, pulso arrítmico. ECG: ondas P de características distintas a la
sinusal, complejos QRS irregulares

167.- ¿Cuál es el diagnóstico más probable?


Fibrilación auricular

Caso 134

Mujer de 65 años, con antecedente de hipertensión arterial sistémica de larga evolución, es valorada
en urgencias por hemiparesia progresiva de hemicuerpo izquierdo que comenzó hace un par de
horas. EF: TA 200/110 mm Hg, FC 85 x’, mirada desviada a la derecha, hemianopsia derecha, afasia
receptiva e hipoestesia izquierda.

168.- ¿Qué tratamiento recomendaría?


Disminución de la TA

Caso 135

A 65-year-old man with diabetes and hypertension without adequate control. Physical examination:
BP 160/100 mm Hg, HR 76 bpm, RR 18/min, the rest of the examination is unremarkable. Creatinine
0.5, Na 136, K 3.8, Cl 100. He takes 100mg per day of Losartan.

169.- Which is the indicated treatment for hypertension control in this patient?
Losartan 100 mg/day+ chlortalidone 50 mg/day

Caso 136

Lactante de 3 meses es traído a consulta de rutina, pero la madre refiere que cierra los ojos frente a
estímulos luminosos y es incapaz de seguir objetos con la mirada. Como antecedentes, el paciente
fue producto de parto prematuro por preeclampsia materna y permaneció hospitalizado durante un
mes. A la oftalmoscopía, se evidencia tejido vascular en el espacio vítreo y vasos tortuosos en el polo
posterior.

170.- ¿En qué rango de edad debe ser capaz de seguir objetos con la mirada?
8-12 semanas

Caso 137
Paciente de 6 años llega a urgencias tras un episodio en el que movía los párpados rápidamente, con
la mirada fija, seguido del desplazamiento de la cabeza lentamente hacia abajo. La madre comenta
que ha presenciado varias situaciones similares, especialmente cuando está sentada. La exploración
física es normal.

171.- ¿Cuál es el hallazgo electroencefalográfico típico en la enfermedad que padece la


paciente?
Descargas punta-onda simétricas

Caso 138

Recién nacido pretérmino, producto de un embarazo patológico, obtenido vía abdominal, es recibido
para reanimación. En el primer minuto, tiene pulso menor a 100x’, flexión de extremidades
superiores, emite sonidos, tiene esfuerzo respiratorio regular y acrocianosis. Conforme transcurre el
tiempo, el esfuerzo respiratorio se debilita, el paciente se torna francamente cianótico e indiferente a
estímulos.

172.- ¿Qué puntaje APGAR tuvo el paciente en las dos determinaciones?


6y3

Caso 139

Hombre de 54 años que padece diabetes, hipertensión y dislipidemia. Acude por un cuadro de 10
meses de evolución caracterizado por dolor de los gastrocnemios al caminar 300 metros y ahora al
caminar 100 metros, al descansar el dolor mejora. A la exploración: signos vitales normales, catarata
bilateral, disminución de pulsos poplíteos, tibiales, pedios.

173.- ¿Cuál es la prueba inicial a realizar para confirmar el diagnóstico?


Índice tobillo-brazo

Caso 140

Niña de 2 meses es traída para regularizar su esquema de vacunación. Desde su nacimiento, la


paciente no ha recibido ninguna inmunización por decisión de los padres. Después de una
exploración física normal y ausencia de sintomatología, usted decide aplicar la vacuna contra
Hepatitis B.

174.- ¿A los cuántos meses de edad se administra la primera dosis de la vacuna contra
varicela?
12
175.- ¿Qué otras vacunas deben aplicarse para alcanzar las necesarias para su edad?
DTPa, Salk, neumocócica, Hib, rotavirus, BCG
176.- ¿Cuántas semanas deben transcurrir entre la segunda y tercera dosis de
hepatitis B?
8

Caso 141

Niño de 4 años es traído a consulta por hemorragia gingival. Estuvo hospitalizado recientemente por
una infección de vías urinarias. A la exploración se encuentra palidez generalizada, talla para la edad
inferior al 5° percentil y petequias en miembros inferiores. Los estudios de laboratorio muestran
leucopenia, neutropenia, trombocitopenia y anemia graves.

177.- ¿Cuál es el diagnóstico más probable?


Anemia aplásica adquirida

Caso 142

Recién nacido a término, femenino, se obtuvo vía abdominal en cesárea de urgencia por
desaceleraciones tardías. Se proporcionó reanimación temprana y se mantuvo en observación. A las
7 horas de vida, aumentó su frecuencia respiratoria y presentó aleteo nasal, tiraje intercostal y
quejido espiratorio. La radiografía de tórax muestra cisuritis y engrosamiento perihiliar.

178.- ¿Cuál es el diagnóstico más probable?


Taquipnea transitoria del recién nacido
179.- ¿Cuál es el tratamiento de elección?
Oxígeno y ayuno

Caso 143
Hombre de 45 años es llevado a urgencias por traumatismo craneoencefálico con
probable fractura cervical. A la exploración: TA 100/60 mm Hg, FC 90 lpm, FR 18 rpm,
S02 90% Glasgow 7, resto irrelevante

180.- ¿Cuál es el dispositivo para la vía aérea indicado en este caso? Mascarilla
laríngea

Caso 144
Neonato femenino, producto de embarazo a término, con madre de 33 años que padece obesidad
grado III, se encuentra en sala de reanimación bajo una fuente de calor. El paciente no respira ni
llora, tiene extremidades en extensión y FC menor a 100 lpm. Se procede a secar, estimular y
posicionar vía aérea; el paciente muestra esfuerzo respiratorio pero la FC se mantiene igual.

181.- ¿Cuál es la acción siguiente según el algoritmo de reanimación neonatal?


Ventilar con presión positiva

Caso 145

Usted recibe a un neonato producto de un parto vaginal. La madre no llevó control prenatal y es
multípara. Después de secar y estimular al paciente, se aprecia fenotipo sugerente de trisomía 21.

182.- ¿Qué alteración gastrointestinal podría padecer el paciente?


Atresia duodenal

Caso 146

Se recibe en consulta a neonato de 3 días de vida por ictericia. El paciente fue producto de embarazo
normoevolutivo y madre primigesta que egresaron sin complicaciones. A la exploración, el paciente
tiene signos vitales normales, se encuentra irritable, ictérico en la cabeza y cuello, y adecuadamente
hidratado.

183.- ¿Cuál es el diagnóstico más probable?


Ictericia fisiológica

184.- ¿Qué recomendación haría?


Fototerapia

Caso 147

Hombre de 22 años acude a urgencias por cefalagia intensa pulsátil, primero unilateral y localizada
en la región retrocular y después holocraneal. El padecimiento inició hace 6 horas, se vuelve más
intenso con el movimiento, le ha causado fotofobia y vómito en dos ocasiones. A la exploración, se
encuentra hipotenso y taquicárdico.

185.- ¿Cuál es el diagnóstico más probable?


Cefalalgia migrañosa
Caso 148

Niño de 8 años llega a urgencias por crisis convulsivas hemiclónicas, precedidas de pérdida brusca de
la visión de algunos secundos. Después de la crisis refirió cefalea intensa. La madre comenta que el
paciente ha presentado cefaleas intensas de difícil control desde los 5 años y vómitos recurrentes. La
exploración física resulta normal.

186.- ¿Cuál es el patrón electroencefalográfico típico de la enfermedad?


Descargas punta-onda occipitales intercríticas
187.- ¿Con qué otra patología se asocia la enfermedad del paciente?
Migraña

Caso 149

Mujer de 23 años inició a los 15 años con cefalea de 6 horas de duración, unilateral, pulsátil
acompañada de náusea, vómito y fotofonofobia. La exploración física es irrelevante.

188.- ¿Cuál es el tratamiento indicado para prevenir ataques subsecuentes?


Amitriptilina
189.- ¿Cuál es el diagnóstico más probable?
Migraña sin aura

Caso 150

Mujer de 75 años con antecedente de tabaquismo, DM2 e hipertrigliceridemia, llega a urgencias por
hemiparesia derecha. EF: TA 140/90 mm Hg, FC 100 x’, alerta, hemiparesia en hemicara y
hemicuerpo derecho, afasia motora.

190.- ¿Cuál es la prioridad del tratamiento?


Rescatar el área de penumbra
191.- ¿Cuál es el diagnóstico más probable?
EVC oclusivo trombótico

Caso 151

Mujer de 25 años, su abuela y una tía tienen enfermedad psiquiátrica desconocida, niega consumo de
drogas. Inició hace 8 meses con alucinaciones visuales que le gritan groserías, lenguaje
desorganizado, no se baña y ha dejado de hacer actividades deportivas que antes realizaba.

192.- ¿Cuál es el tratamiento de elección?


Clozapina
Caso 152

Hombre de 89 años que padece diverticulosis. Acude por 3 horas de evolución con hematoquezia y
pérdida del estado de alerta. A la exploración: TA 70/ 30mm Hg, FC 145 lpm, FR 32 rpm, letárgico,
palidez de piel y tegumentos, sangrado rectal activo, presión de pulso disminuida, retardo en llenado
capilar.

193.- ¿Cuál es el grado de choque que tiene este paciente?


Choque hemorrágico grado IV

Caso 153

Neonato de 28 días es llevado a consulta por hipoactividad, dificultad para la deglución y


constipación desde hace 12 días. Es originaria de una comunidad rural; el nacimiento fue asistido por
una partera y no se inició esquema de vacunación ni se realizó tamiz neonatal. Su hipótesis
diagnóstica es hipotiroidismo congénito.

194.- ¿Cuál es la causa más frecuente de hipotiroidismo congénito?


Disgenesias tiroideas

Caso 154

Hombre de 66 años que padece diabetes e hipertensión; fuma actualmente y tiene índice tabáquico
20 cajetillas/año por periodo de 25 años. Acude a consulta de rutina, niega equivalentes anginosos.
Exploración física irrelevante.

195.- ¿Para cuál de las siguientes enfermedades está indicado el escrutinio?


Aneurisma aórtico abdominal

Caso 155

Mujer de 40 años es valorada por episodios de dolor lancinante paroxístico en hemicara izquierda,
principalmente en región peribucal y cigomática, que comienza al hablar, masticar o sonreír. La
exploración física es normal.

196.- ¿Cuál es el diagnóstico más probable?


Neuralgia trigeminal
Caso 156

A 34-year-old male mentions fatigue and difficulty to obtain and maintain an erection after starting a
medication prescribed by a family physician. He has never had any sexual dysfunction before and he
has two children. He does not recall the diagnosis for which the medication was prescribed.

197.- Which of the following most likely causes the side effect?
Propanolol

Caso 157

Neonato de 4 días llega a urgencias por movimientos anormales rítmicos de las extremidades
superiores que ocurren sin sintomatología acompañante ni desencadenante aparente. Ni el paciente
ni la madre tienen antecedentes patológicos. La exploración física completa, pruebas sanguíneas
generales e imágenes de cráneo resultan normales.

198.- ¿Qué patrón esperaría encontrar en un electroencefalograma?


Patrón alternante theta pointu

Caso 158

Lactante de 2 meses con diagnóstico de trisomía 21 es llevada a consulta por ictericia que persiste
desde el nacimiento. Hace dos semanas se agregó hipocolia e hipoactividad. En la exploración física
se encuentra eutérmica, ictérica y letárgica, con hipotonía axial y hepatomegalia. El grupo sanguíneo
de su padre es O- y el de la madre es O+.

199.- ¿Cuál es el tratamiento indicado para la patología?


Cirugía de Kasai
200.- ¿Cuál es el diagnóstico más probable?
Atresia de vías biliares

Caso 159

Recién nacido, producto de madre con preeclampsia, obtenido vía cesárea a las 37 sdg, se encuentra
flácido, sin esfuerzo respiratorio. Usted realiza los pasos iniciales y el esfuerzo respiratorio se vuelve
regular, FC mayor a 100 lpm, y persiste cianótico.

201.- ¿En qué momento se debería cortar el cordón umbilical?


Después de los primeros 30s de vida
Caso 160

Niño de 9 meses llega a urgencias en estatus epiléptico. Las convulsiones tónico-clónicas se logran
abortar con la primera dosis de diazepam IV. La paciente no tiene antecedentes patológicos, es su
primer evento convulsivo y resulta normal la exploración física

202.- ¿Cuál de los siguientes fármacos está indicado como segunda línea para estatus
epiléptico?
Fenitoína

Caso 161

A 26-year-old woman suffering anxiety. Complains of bilateral, band-like headache for the past three
years, it usually lasts one hour and responds well to pain relivers. General and neurologic
examination without abnormalities.

203.- What is the patient´s most likely diagnosis?


Tension-type headache
204.- Which would be the indicated treatment?
Acetaminophen

Caso 162

Mujer de 44 años, con antecedente de hipertensión arterial sistémica, refiere disnea de esfuerzo y
palpitaciones. A la exploración, los signos vitales son normales. Se ausculta precordio arrítmico,
estertores gruesos en región subescapular, chasquido de apertura en foco mitral, pulsos distales
asíncronos.

205.- ¿Cuál de las siguientes explica la fisiopatología de la enfermedad de la paciente?


Aumento de la presión en la aurícula izquierda

Caso 163

A 69-year-old woman without any important medical history. Her mother presented dementia at 70
years old and her grandmother at 75 years of age. She was brought in because for the past two years
she has had trouble memorizing the date, she has been lost in the grocery store twice and left the
gas open at least once a week. These symptoms have progressed. Physical examination: thin, Mini
Mental 18, ideomotor apraxia, without any motor disfunction. Vitamin B12, folic acid and TSH within
normal levels.

206.- Which is the most probable diagnosis?


Alzheimer´s disease

Caso 164

Mujer de 66 años, con antecedente de tabaquismo desde los 18 años, refiere astenia, pérdida de
peso, accesos de tos, diaforesis nocturna profusa, dolor en cuadrante superior izquierdo abdominal y
saciedad temprana. Hace unos meses tuvo un ECV isquémico en territorio de la arteria cerebral
media izquierda. EF: fiebre, palidez, hepatoesplenomegalia. BH: leucocitos 40,000 células/mcL.

207.- ¿Qué otra alteración esperaría encontrar en estudios paraclínicos?


Hiperuricemia
208.- ¿Cuál es el hallazgo típico de la enfermedad al analizar la médula ósea?

Cromosoma Ph1

Caso 165

Lactante de cuatro meses es llevado a consulta para aplicación de inmunizaciones correspondientes.


Se encuentra asintomático, piel y mucosas con buena coloración e hidratación, en las extremidades
se encuentran pulsos magnus celer (saltones) y se ausculta soplo continuo (en maquinaria) grado III
en foco pulmonar, con irradiación a región infraclavicular izquierda, sin datos de cardiomegalia.

209.- ¿Cuál es el diagnóstico más probable?


Persistencia del conducto arterioso

Caso 166

Hombre de 80 años, con diagnóstico de aneurisma aórtico abdominal en seguimiento tomográfico,


acude a urgencias por dolor súbito e intenso en la extremidad inferior derecha de 6 horas de
evolución. A la exploración, inicia la marcha con esfuerzo y claudica, refiere dolor intenso al movilizar
el pie. El pulso femoral es palpable, pero los pulsos más distales están ausentes.

210.- ¿Cuál es el diagnóstico más probable?


Insuficiencia arterial aguda

Caso 167

Lactante de 4 meses llega a urgencias por eventos de llanto y movimientos de flexión, extensión o
alternantes que duran aproximadamente 2 minutos. Es incapaz de sentarse solo y de sostener la
cabeza. Presenta talla baja para la edad y múltiples angiofibromas faciales.

211.- ¿Qué síndrome epiléptico presenta el paciente?


Espasmos infantiles

Caso 168

Mujer de 28 años, con antecedente de epilepsia desde los 14 años. Actualmente cursa con embarazo
de 6 SDG y no toma ningún antiepiléptico; está hospitalizada por crisis convulsivas tónico-clónico
generalizadas. La exploración física y los estudios de laboratorio son irrelevantes.

212.- ¿Cuál es el tratamiento de elección para las crisis convulsivas de esta paciente?
Lamotrigina

Caso 169

Hombre de 85 años tiene diagnóstico de hipertensión arterial sistémica. Acude por un día de
evolución con cefalea y pérdida del estado de alerta. A la exploración física: TA 240/140 mm HG, FC
76 lpm 18 rpm, Glasgow 3. En la TAC de cráneo hay hemorragia en el putamen.

213.- ¿Cuál es el tratamiento de elección en este caso?


Nitroprusiato de sodio

Caso 170

Niña de 7 años llega a urgencias por disnea después de comer un durazno. Tiene antecedente de
alergia al látex. EF: taquicardia, taquipnea, saturación de Hb 89%, eritema y edema perioral, palatino
y faríngeo.

214.- ¿Cuál es el tratamiento?


Epinefrina
Caso 171

Neonato a término obtenido vía abdominal por preeclampsia materna severa. Al primer minuto, se
encuentra cianótico, pobre esfuerzo respiratorio, con hipotonía generalizada y FC menor a 100. El
paciente requirió reanimación e intubación. En su primer hora de vida, se mantenía poco reactivo a
estímulos e hipotenso.

215.- ¿Cuál es el tratamiento inicial indicado?


Cloruro de calcio IV

Caso 172

Mujer de 26 años, padece ansiedad. Acude por un cuadro de tres años de evolución caracterizado por
cefalea holocraneana de una hora de duración que mejora con analgésicos y lo asocia a episodios de
estrés. La exploración general y neurológica es normal.

216.- ¿Cuál es el diagnóstico más probable?


Cefalea tensional

Caso 173

Hombre de 61 años, con antecedente de hipertensión y cirugía de reemplazo valvular mitral, ingresa
a urgencias por hemiparesia izquierda progresiva. EF: TA 160/100 mm Hg, FC 104x’, arritmia cardiaca
con soplo y clic en foco mitral, hemiparesia izquierda y Babinski.

217.- ¿Cuál es el diagnóstico más probable?


Embolia cerebral

Caso 174

Recién nacido a término es valorado en la sala de reanimación. El paciente se obtuvo vía abdominal
por desaceleraciones tardías en las evaluaciones cardiotocográficas y se observó meconio en el
líquido amniótico. Después de un minuto de vida, el paciente se encuentra con acrocianosis, tiraje
intercostal apenas visible, mínimo aleteo nasal y quejido respiratorio audible con el estetoscopio.

218.- ¿Qué significa la presencia de meconio en el líquido amniótico?


Estimulación de los sistemas simpático y parasimpático
Caso 175

Neonato femenino de 2 días se encuentra hospitalizada por distensión abdominal, rechazo al


alimento y ausencia de evacuaciones. El estudio radiológico realizado reporta zona de estenosis en
región recto-sigmoidea con dilatación considerable del colon proximal.

219.- ¿Cuál es la fisiopatología de la enfermedad?


Ausencia de plexos mientérico y submucoso

Caso 176

Mujer de 65 años consulta por fiebre, astenia, pérdida de peso, diaforesis nocturna y tumoración en la
región cervical anterior izquierda de 3 meses de evolución. Hace varios días, comenzó con accesos de
tos seca y disfonía. EF: fiebre, adenopatía cervical anterior de 4cm de diámetro indurada y fija,
esplenomegalia.

220.- ¿Cuál es la fisiopatología de la enfermedad?


Expansión clonal progresiva de células B

Caso 177

Niño de 7 años es traído al servicio de urgencias con hemorragia gingival, fiebre y disminución del
estado de alerta. El padecimiento inició hace diez días con tos, esputo abundante y fiebre, por lo que
recibió tratamiento antibiótico (no especificado). Parecía mejorar hasta hace un día que inició otro
episodio febril y epistaxis profusa. A la exploración se encuentra con Glasgow de 9 puntos, tensión
arterial de 75/40 mmHg, fiebre de 39.3°C, un coágulo en cavidad nasal derecha, numerosas
equimosis en miembros pélvicos y palidez generalizada. El laboratorio reporta anemia, neutropenia y
trombocitopenia severas.

221.- ¿Cuál es el diagnóstico más probable?


Síndrome de falla medular

Caso 178

Hombre de 80 años, padece obesidad e hipertensión. Acude a urgencias por pérdida de la fuerza
muscular en hemicuerpo izquierdo desde hace 1 ½ hora. A la exploración física: TA 160/90 mm Hg,
FC 90 lpm, FR 18 rpm, S02 94%, ruidos cardiacos sin alteraciones, consciente, hemiparesia izquierda
y parálisis facial central. Laboratorios normales.

222.- No hay hemorragia ni tumor. ¿Cuál es el tratamiento de elección?


Trombolisis

Caso 179

A 23-year-old female arrives at the physician after one episode of severe anxiety, palpitations, chest
pain, and shortness of breath. Physical examination is unremarkable. He denies drug use and an ECG
shows no abnormalities.

223.- Which would be the most appropriate treatment?


Paroxetine

Caso 180

Hombre de 40 años, maratonista, acude a urgencias por palpitaciones. A la exploración física: TA


120/70 mm Hg, FC 150 lpm, FR 20 rpm, no hay datos de falla cardiaca. En el electrocardiograma
encuentras taquicardia sinusal (150 lpm) con complejos QRS estrechos.

224.- ¿Cuál es el diagnóstico electrocardiográfico?


Taquicardia supraventricular

Caso 181

Mujer de 22 años acude para seguimiento de displasia cervical. Tiene historia de múltiples parejas
sexuales con uso irregular del condón y tricomoniasis resuelta hace un año. Consume tabaco y
alcohol al menos una vez por semana, tiene múltiples tatuajes y perforaciones.

225.- ¿Cuál es el principal factor de riesgo para la enfermedad que padece?


Infección por VPH

Caso 182

Mujer de 24 años, con embarazo gemelar monocorial de 30sdg, se encuentra hospitalizada por
amenaza de parto prematuro con contracciones regulares, dilatación de 2cm y borramiento 30%. El
ultrasonido reporta restricción del crecimiento de un gemelo y oligohidrammnios.

226.- ¿Cuál es el principal efecto adverso del medicamento seleccionado?


Hipotensión
Caso 183

Mujer de 45 años previamente sana y con paridad satisfecha. Acude por un cuadro de tres años de
evolución caracterizado por hiperpolimenorrea. A la exploración se encuentra con palidez de
tegumentos. Hb 9.8 VCM 76. Ultrasonido pélvico con miomatosis uterina de grandes elementos.

227.- ¿Cuál es el tratamiento de elección en esta paciente?


Histerectomía

Caso 184

Mujer de 26 años refiere disuria, dispareunia y prurito vulvar. Desde hace una semana, notó
secreción vaginal fétida y espumosa color verde. Como antecedentes, ha tenido múltiples parejas
sexuales y utiliza condón de forma irregular.

228.- ¿Cuál es la etiología más probable de la enfermedad?


Trichomonas vaginalis

Caso 185

Mujer embarazada de 22 años de edad con 14sdg acude a consulta porque su hermano pequeño fue
recientemente diagnosticado con rubéola. La paciente desconoce si se ha aplicado la vacuna contra
el virus, no tiene ningún síntoma y presenta un ultrasonido transvaginal que confirma la edad
gestacional y se reporta sin alteraciones.

229.- ¿En qué periodo es mayor el riesgo de infección del feto?


Después de la semana 36

Caso 186

Mujer de 65 años, con antecedente de cáncer endometrial estadio II, acude a consulta de
seguimiento y a nueva toma de biopsia.

230.- ¿Cuál es la variante histológica más común de la enfermedad?


Endometroide
231.- ¿Qué implica el estadio de la enfermedad de la paciente?
Invasión del estroma cervical sin extensión fuera del útero
Caso 187

Paciente de 34 años, primigesta con 38sdg por FUM, refiere dolor abdominal cólico en episodios de
un minuto cada cinco minutos. Hace doce horas se colocó una perla intravaginal por indicación
médica. EF: signos vitales normales, fondo uterino 37cm, producto cefálico, longitudinal, dorso
izquierdo, encajado. Frecuencia cardiaca fetal normal, movimientos espontáneos. Tacto vaginal:
cérvix posterior, 2cm de dilatación, borramiento 30%, membranas íntegras.

232.- ¿Cuál es el siguiente paso en el abordaje de la paciente?


Seguimiento ambulatorio

Caso 188

Mujer de 30 años, con antecedente de 4 gestaciones previas resueltas sin complicaciones, cursa con
enfermedad trofoblástica gestacional diagnosticada por USG. La paciente no desea embarazarse
más.

233.- En el caso específico de la paciente, ¿qué manejo está recomendado?


Histerectomía

Caso 189

Mujer de 27 años acude a control prenatal con embarazo de 25 sdg. AGO: G2, P0, C1 por producto
macrosómico, sin enfermedades relevantes. La paciente tiene antecedentes familiares de DM2 e
hipertensión arterial sistémica y ha sido obesa desde la adolescencia. Tiene una química sanguínea
previa realizada durante el primer trimestre con glucemia en ayuno normal.

234.- ¿A qué edad gestacional se debe realizar esta prueba?


Entre la semana 24 y 28

Caso 190

Mujer de 24 años, con embarazo de 33sdg, es valorada en consulta por palpitaciones y disnea. En las
últimas semanas, el útero ha crecido con una tasa mayor de lo usual. A la exploración, se encuentra
taquicárdica, taquipnéica, con fondo uterino de 47cm y producto único vivo longitudinal, con dorso a
la derecha y FCF de 150. El ultrasonido reporta ILA de 32.

235.- ¿Qué prueba complementaria de tamizaje debe indicar para la paciente?


Prueba de tolerancia a la glucosa
236.- ¿Qué indica el índice de líquido amniótico?
Polihidramnios severo
237.- ¿Cuál es la etiología más probable del diagnóstico?
Atresia esofágica
238.- ¿Cuál es el tratamiento indicado?
Drenaje de líquido amniótico

Caso 191

Adolescente de 17 años acude consulta de control prenatal. La paciente sospechó de su embarazo


después de tres semanas de amenorrea y lo comprobó con una prueba sanguínea. No tiene
antecedentes patológicos ni sintomatología fuera de náuseas y vómitos matutinos. Ha tenido tres
parejas sexuales en el último año, utilizaba método de barrera de forma irregular y nunca había
acudido a una valoración ginecológica. Consumía alcohol de forma semanal y tabaco cada tercer día
antes de saberse embarazada.

239.- ¿Qué efecto fetal puede tener el consumo de tabaco?


Bajo peso al nacer

Mujer de 28 años, multigesta, actualmente con embarazo de 11sgd, acude a urgencias por dolor
abdominal súbito, constipación y vómito. El dolor es de carácter cólico, difuso, y se irradia a la
espalda. A la exploración, la paciente se encuentra ansiosa, hipotensa, taquicárdica, con mucosas
secas, distensión abdominal importante y dolor generalizado a la palpación. Las pruebas de
laboratorio reportan leucocitosis y hemoconcentración.

240.- ¿Qué prueba de imagen solicitaría?


Radiografía de pie y decúbito abdominal

Caso 193

Mujer de 35 años, multípara y con embarazo de 27 sdg, llega a urgencias por dolor epigástrico
generalizado tipo cólico que se irradia al flanco. La paciente ha vomitado más de 5 veces, es incapaz
de mantenerse sentada y refiere preocupación por el producto. Esta es la segunda vez que sufre un
dolor similar, sin embargo, la vez previa se autolimitó después de un par de horas. EF: taquicardia y
taquipnea, temperatura 37.8°C, hiperestesia en el área descrita e intolerancia a la palpación
profunda, rebote negativo.

241.- ¿Qué circunstancias en el embarazo hacen propensa a la paciente para la


enfermedad que padece?
Mayor cantidad de sal biliar circulante

Caso 194

Mujer de 22 años, primigesta, con embarazo de 34 sdg, llega a urgencias por cefalalgia y acúfenos.
Tiene los resultados de un EGO que muestran proteinuria +/+++. A la exploración, TA 145/100
mmHg, FC 90x’ y FCF 140x’. Mientras se dispone a realizar el registro cardiotocográfico, la paciente
sufre una crisis convulsiva tónico clónica generalizada.

242.- ¿Cuál es un factor de riesgo para el padecimiento?


Obesidad

Caso 195

Mujer de 25 años, con embarazo de 17sdg, llega a urgencias por salida transvaginal de líquido con
trazas hemáticas. Durante la exploración, expulsa espontáneamente al producto. La mujer refiere
que es la segunda ocasión que le ocurre, sólo que la primera vez tenía 23sdg.

243.- ¿Cómo se realiza el diagnóstico?


Historia clínica

Caso 196

Mujer de 30 años, gesta 3 para 0 abortos 3, acude a consulta de seguimiento posterior a legrado
uterino instrumentado. La paciente está muy afectada por la tercer pérdida y desea someterse a
estudios diagnósticos para evitar otro evento similar. Todos los abortos han ocurrido en el segundo
trimestre.

244.- ¿Qué factores originan pérdidas con mayor frecuencia en el segundo trimestre?
Uterinos

Caso 197
Mujer de 37 años acude a consulta por incapacidad de embarazarse después de un año y medio de
relaciones sexuales regulares sin protección. La paciente no tiene antecedentes patológicos, nunca
ha estado embarazada y ha utilizado preservativo como método anticonceptivo desde que comenzó
su vida sexual. La exploración ginecológica es normal.

245.- ¿Cuál es el factor causal más probable del padecimiento?


Masculino

Caso 198

Mujer embarazada llega a urgencias con pérdida del estado de alerta, hipotensión y taquicardia. Su
familiar refiere que comenzó hace media hora con dolor abdominal intenso y hemorragia vaginal. A la
exploración, el fondo uterino es de 39cm y los movimientos fetales están disminuidos.

246.- ¿Cómo espera encontrar el útero?


Hipertónico
247.- ¿Cuál es el diagnóstico más probable?
Abruptio placentae

Caso 199

Mujer de 71 años, postmenopáusica y con antecedente de 7 partos, acude a consulta por


poliaquiuria, urgencia, nicturia y enuresis nocturna. Los síntomas comenzaron hace un mes, han sido
progresivos y han afectado considerablemente la calidad de vida de la paciente. Presenta resultado
de EGO dentro de parámetros normales.

248.- ¿Qué elemento de la micción está probablemente afectado?


Contracción del detrusor

Caso 200

Adolescente de 14 años de edad es traída a consulta por su madre; comenta que desde hace seis
meses su hija cursa con amenorrea secundaria y ha disminuido su peso de 65 a 32 kg. La paciente
niega inicio de vida sexual y muestra aversión a la misma debido a que teme subir de peso. A la
exploración se encuentra con un peso de 33 kg, talla de 148 cm, palidez generalizada, prominencias
óseas evidentes, cabello fácilmente desprendible y piel seca.

249.- ¿Cuál es la causa más probable de la amenorrea?


Disminución de FSH y LH
250.- ¿Qué pruebas diagnósticas requiere para completar el abordaje?
Perfil lipídico y electrolitos
Hombre de 57 años se presenta por síntomas de urgencia y nicturia. También refiere molestias opresivas y
dolor perineal que no sabe localizar con exactitud. Señala que lleva más de medio año con el malestar,
presentado periodos con mayor molestia y otros en los que es rara la presencia de síntomas.
Ya ha recibido tratamiento para infección de vías urinarias en algunas ocasiones con escasa mejoría y en el
EGO hay bacteriuria persistente.
251.- ¿Cuál es el diagnostico mas probable?
Prostatitis bacteriana cronica
252.- ¿Como se confirma el diagnostoco?
Bacterias en liquido prostatico
253.- ¿Cuánto tiempo tiene que durar el tratamiento antimicrobiano?
Minimo 6 semanas

Mujer de 61 años, con antecedente de diabetes mellitus tipo 2 desde hace 18 años y retinopatía diabética
proliferativa tratada con láser. En tratamiento irregular a base de hipoglucemiantes orales. Acude a consulta
porque desde hace unos meses padece dolor urente en la planta de los pies. Además edema de miembros
inferiores al estar de pie. A la exploración FC 83 Ipm, FR 18 rpm, temperatura 36.5°C, TA 140/80 mm Hg.
Destaca la presencia de edema en extremidades inferiores (+++) hasta el tercio medio de la pantorrilla. Tira
reactiva de orina con proteínas ++++, glucometría capilar 280 mg/dl.
254.- ¿Cuál es el diagnostico mas probable?
Sindrome nefrotico

Hombre de 65 años de edad hipertenso, acude a oftalmólogo para evaluación de rutina.


A la exploración física se observan ambos ojos sin alteraciones, la agudeza visual es de
20/20 en ambos ojos, fondo de ojo con relación excavación/papila (E/P) de 0.7, presión
intraocular de 25 mm g en ambos ojos

255.- ¿Cuál es la presion intraoptica considerada normal?


10-21 mmHg
256.- ¿Cuál es el diagnostico mas probable?
Glaucoma de angulo abierto
257.- ¿Cuál es el primer paso mas adecuado en el tratamiento de este paciente?
Manitol

Hombre de 45 años de edad con diagnósticos de hipertensión arterial sistémica en


tratamiento con tiazidas y psoriasis en placa, manejada con esteroides y calcipotriol
tópicos. Acude a consulta ya que durante 2 años ha presentado periodos en los cuales
no necesita dormir, trabaja más de 18 horas al día y presenta prácticas sexuales de
riesgo, estos episodios se alternan con múltiples periodos en los cuales no disfruta de ir
al trabajo, disminuye su apetito, se siente triste y presenta
insomnio por despertar precoz.
258.- ¿Cuál de los siguientes farmacos es el indicado?
carbamazepina

Mujer de 35 años con diagnóstico de hipotiroidismo autoinmune inició padecimiento hace 8 semanas con
ronchas pruríticas en piernas y muslos. Las lesiones aparecen y desaparecen en un lapso de tiempo mayor a 48
horas y dejan cicatriz. A la exploración, se observa la dermatosis descrita anteriormente, las ronchas no
desaparecen a la digitopresión
259.- ¿Cuál es el diagnostico mas probable?
Vasculitis urticariforme
260.- ¿Cuál es el mecanismo fisiopatologico de esta enfermedad?
Hipersensibilidad tipo lll

Mujer de 60 años de edad con antecedente de tabaquismo por 30 años. Se presenta a


la consulta por dolor progresivo en ambas manos, muñecas y tobillos de un año de
evolución, rigidez al despertar que dura aproximadamente una hora. A la exploración
física se encuentra inflamación simétrica de articulaciones interfalángicas proximales,
metacarpofalángicas, las propias del carpo y tobillos. Signo de Morton positivo en
interfalángicas, metacarpofalángicas y tobillos. En la radiografía de manos se observa
inflamación de partes blandas, osteopenia y erosiones articulares.
261.- ¿Cuál es el tratamiento de primera linea?
Metotrexate. Poliartritis

Mujer de 28 años, G:3, con antecedente de parto pretérmino en embarazo anterior. Acude al
servicio de urgencias con embarazo de 30.3 Semanas de Gestación por contracciones uterinas
irregulares, dolorosas; a la exploración física sus signos vitales son: TA 120/80, FC100 1pm, FR
18. A la exploración útero gestante con producto único, vivo, fondo uterino de 29 cm. Se palpan
contracciones con duración de 30 segundos, una cada 10 minutos, Frecuencia Cardiaca Fetal de
145 Ipm, cérvix posterior con dilatación de 3 cm, borramiento 20%
262.- ¿Cuál es el mejor siguiente paso?
Medicion de fibronectina y longitud cervical
263.- Si los resultados del estudio solicitado revelaran longitud cervical de 22 mm y
fibronectina fetal positiva, ¿Cuál sería el mejor tratamiento en ésta paciente?
Aplicación de indometacina + dexametasona

Hombre de 40 años de edad, obeso. Acude a consulta por 1 cuadro de 11


meses de evolución, caracterizado por odinofagia y tos matutina, pirosis,
regurgitación, disfagia Se decide dar manejo con inhibidor de bomba de
protones y realizar endoscopía, en la que se observa úlcera que ocupa toda
la circunferencia de la mucosa esofágica, se toman múltiples biopsias
264.- ¿Qué porcentaje de pacientes con esofagitis desarrollan esofago de barret?
20%
265.- ¿Cuál es la complicacion de la entidad de base de este paciente?
Estenosis peptica
266.- ¿Qué grado de clasificacion de los angeles presenta este paciente?
Grado D
Niña de 10 años que es llevada a consulta para seguimiento porque su índice de masa
corporal entre las percentilas 85 y 95 para su edad. En las tres últimas consultas
presentó una tensión arterial sistólica entre las percentilas 90 y 95.
267.- ¿Cómo se clasifica el índice de masa corporal de acuerdo a lapercentila en la que
se ubica a la paciente?
sobrepeso
268.-¿Cómo se clasifica la tensión arterial de acuerdo a la percentila erla que se ubica a la paciente?
Prehipertension
269.-¿Cuál es la principal causa de hipertension arterial sistemica en menores de 12 años?
Anormalidades del parenquima renal

Mujer de 50 años de edad con antecedente de tromboembolia pulmonar hace 2 años.


Acude a urgencias por un cuadro que inició hace 2 horas, caracterizado por disnea
súbita, dolor torácico y tos. Signos vitales: TA 110/70 mm Hg, FC 130 Ipm, FR 25 rpm,
T 36.5C. A la exploración física dolor a la palpación en pantorrilla izquierda. La
radiografía de tórax muestra condensación parenquimatosa triangular de base pleural
derecha.
270.-¿Cuál es la alteracion electrocardiografica mas frecuente en pacientes con esta enfermedad?
Taquicardia sinusal
271.-¿Cuál es el estdio diagnostico de eleccion?
Tac helicoidal con contraste
272.-¿Cuál es el tratamiento mas adecuado para este paciente?
Anticoagulacion

Niño de 4 años de edad es traído a consulta por hematuria. Al interrogatorio, la madre revela que ha sufrido
infecciones de vías urinarias en varias ocasiones, el último cuadro fue tratado con cefuroxima durante 14 días.
Actualmente el niño orina con mucha frecuencia, no controla esfínteres. La orina es color rojizo, con coágulos
pequeños
273.-¿Cuál de las siguientes opciones es apropiada para este caso?
Referir a 3er nivel
274.-¿Cuál es la cuausa mas frecuente de infecciones de vias urinarias en este paciente?
Reflujo vesicouretral

Hombre de 42 años que padece obesidad mórbida. Acude porun cuadro de un año
caracterizado por somnolencia diurna, ronquidos nocturnos y ha tenido un accidente
automovilístico por quedarse dormido. A la exploración: TA 140 mm Hg, FC 9 Ipm, S02
92%, cuello corto y ancho, obesidad centrípeta.
275.-¿Cuál es el diagnostico mas probable?
Sindrome de apnea / hipopnea obstructiva del sueño
Mujer de 25 años padece diabetes mellitus tipo 1 desde hace 7 años e hipotiroidismo autoinmune desde hace
3 años, recibe tratamiento con insulina y levotiroxina. Acude a consulta por cuadro de dos meses de evolución
caracterizado por adinamia, anorexia, pérdida de peso, náuseas, vómitos y oscurecimiento de la piel en zonas
no expuestas al sol. Laboratorio: cortisol plasmático basal 2.8 mg/dl.
276.-¿Qué diagnostico puede integrarse?
Sx poliglandular autoinmune tipo ll

Hombre de 82 años se encuentra hospitalizado por fracturasubtrocantérica de fémur


derecho. Desde su ingreso, ha presentado agitación psicomotriz, habla incoherente,
agresividad hacia familiares y médicos y fluctuaciones del estado de alerta con
episodios de hipoactividad. A la exploración física: desorientado, habla incoherente,
irritable y con acortamiento de la pierna derecha. Paraclínicos: Na 137, Hb14,
Leucocitos 6,500 Neu 65%, examen general de orina y placa de tórax normal
277.-¿Qué tratamiento debe iniciarse?
Terapia hormonal con estrogenos

Recién nacido masculino de 38 semanas de edad gestacional, obtenido por parto eutócico. A la exploración
física: peso 3200 g, talla 54 cm, leve aleteo nasal y cianosis perioral; Apgar de 9 (al inicio y a los 5 minutos).
Adecuado apego al seno materno, con micciones y deposiciones al corriente. Realizas radiografía en la cual se
observas tumoración mediastinal con niveles hidroaéreos superpuesta a la sombra cardiaca
278.-¿Cuál es el diagnostico mas probable?
Hernia de Morgagni

Hombre de 32 años con epilepsia desde los 13 años y buenapego farmacológico es llevado a urgencias por
presentar 3 crisis convulsivas de 10 minutos de duración cada una. Sin recuperación del estado de alerta entre
ellas. Además, desde hace tres días tiene fiebre v cefalea. Se encuentran signos vitales normales, Glasgow 8,
miosis y rigidez de nuca. Electrolitos séricos y gasometría arterial normales, leucocitos 22,550, neutrófilos 85%
279.-¿Cuál es el diagnostico mas probable?
Estatus epileptico convulsivo
279.-¿Cuál es el primer paso en el tx?
Asegurar via aerea

Hombre de 32 años de edad, es traído al servicio de urgencias


tras tener un accidente automovilístico en el que viajaba sin
cinturón de seguridad. Al ingreso se encuentra con alteración del
estado de alerta. Signos vitales TA 80/60, FR 24 rpm,
disneico, FC 120 lpm, T: 36°C. A la exploración física se observa
ingurgitación de venas del cuello, pulso débil y ruidos cardiacos
de intensidad disminuida. En el electrocardiograma la única
anormalidad es una disminución del voltaje de todas
las derivaciones.
280.-¿Cuál es el tratamiento mas adecuado para este paciente?
Puncion en region subxifoidea

También podría gustarte